Scrigroup - Documente si articole

     

HomeDocumenteUploadResurseAlte limbi doc
BulgaraCeha slovacaCroataEnglezaEstonaFinlandezaFranceza
GermanaItalianaLetonaLituanianaMaghiaraOlandezaPoloneza
SarbaSlovenaSpaniolaSuedezaTurcaUcraineana

AdministrationAnimalsArtBiologyBooksBotanicsBusinessCars
ChemistryComputersComunicationsConstructionEcologyEconomyEducationElectronics
EngineeringEntertainmentFinancialFishingGamesGeographyGrammarHealth
HistoryHuman-resourcesLegislationLiteratureManagementsManualsMarketingMathematic
MedicinesMovieMusicNutritionPersonalitiesPhysicPoliticalPsychology
RecipesSociologySoftwareSportsTechnicalTourismVarious

Critical Reasoning

grammar



+ Font mai mare | - Font mai mic



Critical Reasoning

Critical reasoning questions appear in the Verbal section of the GMAT-exam The Verbal section uses multiplechoice questions to measure your ability to read and comprehend written material to reason and evaluate arguments and to correct written material to conform to standard written English Because the Verbal section includes content from a variety of topics you may be generally familiar with some of the material however, neither the passages nor the questions assume knowledge of the topics discussed Critical reasoning questions are intermingled with reading comprehension and sentence correction questions throughout the Verbal section of the exam



You will have 75 minutes to complete the Verbal section or about 1 minutes to answer each question Although critical reasoning questions are based on written passages these passages are shorter than reading comprehension passages They tend to be 1ess than 100 words in 1en~th and generally are followed by one or two questions For these questions you will see a split computer screen The written passage will remain visible as each question associated with that passage appears in turn on the screen You will see only one question at a time

Critical reasoning questions are designed to test the reasoning skills involved in(1)making arguments (2)evaluating arguments and(3)formulating or evaluating a plan of action The materials on which questions are based are drawn from a variety of sources The GMAT-test does not suppose any familiarity with the subject matter of those materials

In these questions you are to analyze the situation on which each question is based and then select the answer choice that most appropriately answers the question Begin by reading the passages carefully, then read the five answer choices If the correct answer is not immediately obvious to you see whether you can eliminate some of the wrong answers Reading the passage a second time may be helpful in illuminating subtleties that were not immediately evident

Answering critical reasoning questions requires no specialized knowledge of any particular field you don't have to have knowledge of the terminology and conventions of formal logic The sample critical reasoning questions in this chapter illustrate the variety of topics the exam may cove, the kinds of questions it may ask, and the level of analysis it requires

The following pages describe what critical reasoning questions are designed to measure and present the directions that will precede questions of this type Sample questions and explanations of the correct answers follow

1 What is Measured

Critical reasoning questions are designed to provide one measure of your ability to reason effectively in the following areas

Argument Construction

Questions in this category may ask you to recognize such things as the basic structure of an argumentproperly drawn conclusions underlying assumptions well-supported explanatory hypotheses or parallels between structurally similar arguments

Argument Evaluation

These questions may ask you to analyze a given argument and to recognize such things as factors that would strengthen or weaken the given argument reasoning errors committed in making that argument or aspects of the method by which the argument proceeds

Formulating and evaluating a plan of action

This type of question may ask you to recognize such things as the relative appropriateness effectiveness or efficiency of different plans of action factors that would strengthen or weaken the prospects of success for a proposed plan of action or assumptions underlying a proposed plan of action

2 Test-Taking Strategies for Critical Reasoning Questions

Read very carefully the set of statements on which a question is based

Pay close attention to

what is put forward as factual information

what is not said but necessarily follows from what is said

what is claimed to follow from facts that have been put forward and

how well substantiated are any claims that a particular conclusion follows from the facts that have been put forward

In reading the arguments it is important to pay attention to the logical reasoning used

the actual truth of statements portrayed as fact is not important

Identify the conclusion

The conclusion does not necessarily come at the end of the text it may come somewhere in

the middle or even at the beginning. Be alert to dues in the text that an argument follows logically from another statement or statements in the text

Determine exactly what each question asks

You might find it helpful to read the question first before reading the material on which it is based don't assume that you know what you will be asked about an argument An argument may have obvious flaws and one question may ask you to detect them But another question may direct you to select t11e one answer choice that does NOT describe a flaw in the argument

Read all the answer choices carefully

Do not assume that a given answer is the best without first reading all the choices

3 The Directions

These are the directions you will see for critical reasoning questions when you take the GMAT-test. If you read them carefully and understand them dearly before going to sit for the exam you will not need to spend too much time reviewing them when you are at the test center and the exam is under way

For this question select the best of the answer choices given

4 Critical Reasoning Sample Questions

For these questions, select the best of the answer choices given

1. Which of the following best completes the passage below?

In a survey of job applicants two-fifths admitted to being at least a little dishonest However, the survey may underestimate the proportion of job applicants who are dishonest because 

(A) some dishonest people taking the survey might have claimed on the survey to be honest

(B) some generally honest people taking the survey might have claimed on the survey to be dishonest

(C) some people who claimed on the survey to be at least a little dishonest may be very dishonest

(D) some people who claimed on the survey to be dishonest may have been answering honestly

(E) some people who are not job applicants are probably at least a l le dishonest

2.Increases in the level of high-density lipoprotein (HDL)in the human bloodstream lower bloodstream cholesterol levels by increasing the body s capacity to rid itself of excess cholestero1 Levels of HDL in the bloodstream of some individuals are significantly increased by a program of regular exercise and weight reduction

Which of the following can be correctly inferred from the statements above?

(A) Individuals who are underweight do not run any risk of developing high levels of cholesterol in the bloodstream

(B) Individuals who do not exercise regularly have a high risk of developing high levels of cholesterol in the bloodstream late in lire

(C) Exercise and weight reduction are the most effective methods of lowering bloodstream cholesterol levels in humans

(D) A program of regular exercise and weight reduction lowers cholesterol levels in the bloodstream of some individuals

(E) Only regular exercise is necessary to decrease cholesterol levels in the bloodstream of individuals of average weight

3. A cost effective solution to the problem of airport congestion is to provide high-speed ground transportation between major cities lying 200 to 500 miles apart The successful implementation of this plan would cost far less than expanding existing airports and would also reduce the number of airplanes clogging both airports and airways

Which of the following if true could proponents of the plan above most appropriately cite as a piece of evidence for the soundness of their plan?

(A) An effective high-speed ground--transportation system would require major repairs to many highways and mass-transit improvements

(B) One-half of all departing flights In the nation's busiest airport head for a destination In a major city 225 miles away

(C) The majority of travelers departing from rural airports are flying to destinations in cities over 600 miles away

(D) Many new airports are being built in areas that are presently served by high-speed ground-transportation systems

(E) A large proportion of air travelers are vacationers who are taking long-distance flights

The price the government pays for standard weapons purchased from military contractors is determined by a pricing method called historical costing Historical costing allows contractors to protect their profits by adding a percentage increase based on the current rate of inflation to the previous years contractual price

Which of the following statements if true is the best basis for a criticism of historical costing as an economically sound pricing method for military contracts?

(A) The government might continue to pay for past inefficient use of funds

(B) The rate of inflation has varied considerably over the past twenty years

(C) The contractual price w-II be greatly affected by the cost of materials used for the products

(D) Many taxpayers question the amount of money the government spends on military contracts

(E) The pricing method based on historical costing might not encourage the development of innovative weapons

5. Which of the following best completes the passage below? Established companies concentrate on defending what they already have Consequently, they tend not to be innovative themselves and tend to underestimate the effects of the innovations of others The clearest example of this defensive strategy is the fact that 

(A) ballpoint pens and soft-tip markers have eliminated the traditional market for fountain pens clearing the way for the marketing of fountain pens as luxury or prestige items

(B) a highly successful automobile was introduced by the same company that had earlier introduced a model that had been a dismal failure

(C) a once-successful manufacturer of slide rules reacted to the introduction of electronic calculators by trying to make better slide rules

(D) one of the first models of modern accounting machines designed for use in the banking industry, was purchased by a public library as well as by banks

(E) the Inventor 0f a commonly used anesthetic did not intend the product to be used by dentists who currently account for almost the entire market for that drug

Since the mayors publicity campaign for Greenville's bus service began six months ago morning automobile traffic into the midtown area of the city has decreased seven percent During the same period there has been an equivalent rise in the number of persons riding buses into the midtown area Obviously, the mayor's publicity campaign has convinced many people to leave their cars at home and ride the bus to work

Which of the following if true casts the most serious doubt on the conclusion drawn above?

(A) Fares for all bus routes in Greenville have risen an average of five percent during the past six months

(B) The mayor of Greenville rides the bus to City Hall in the city s midtown area

(C)  Road reconstruction has greatly reduced the number of lanes available to commuters in major streets leading to the midtown area during the past six months

(D) The number of buses entering the midtown area of Greenville during the morning hours is exactly the same now as it was one year ago

(E) Surveys show that longtime bus riders are no more satisfied with the Greenville bus service than they were before the mayors publicity campaign began

7. A researcher discovered that people who have low levels of immune-system activity tend to score much lower on tests of mental health than do people with normal or high immune-system activity The researcher concluded from this experiment that the immune system protects against mental illness as well as against physical disease

The researchers conclusion depends on which of the following assumptions?

(A) High immune-system activity protects against mental illness better than normal immune-system activity does

(B) Mental illness is similar to physical disease in its effects on body systems

(C) People with high immune-system activity cannot develop mental illness

(D) Mental illness does not cause people's immune-system activity to decrease

(E) Psychological treatment of mental illness is not as effective as is medical treatment

8. It is true that it is against international law to sell plutonium to countries that do not yet have nuclear weapons But if United States companies do not do so companies in other countries will

Which of the following is most like the argument above in its logical structure?

(A) It is true that it is against the police department's policy to negotiate with kidnappers But if the police want to prevent loss of lire they must negotiate in some cases

(B) It is true that it is illegal to refuse to register for military service But there is a long tradition in the United States of conscientious objection to serving in the armed forces

(C) It is true that it is illegal for a government official to participate in a transaction in which there is an apparent conflict of interest But if the facts are examined carefully, it will clearly be seen that there was no actual conflict of interest in the defendant's case

(D) It is true that it is against the law to burglarize people's homes But someone else certainly would have burglarized that house if the defendant had not done so first

(E) It is true that company policy forbids supervisors to fire employees without two written warnings. But there have been many supervisors who have disobeyed this policy

Extinction is a process that can depend on a variety of ecological geographical and physiological variables These variables affect different species of organisms in different ways and should therefore yield a random pattern of extinctions However, the fossil record shows that extinction occurs in a surprisingly definite pattern with many species vanishing at the same time

Which of the following if true forms the best basis for at least a partial explanation of the patterned extinctions revealed by the fossil record?

(A) Major episodes of extinction can result from widespread environmental disturbances that affect numerous different species

(B) Certain extinction episodes selectively affect organisms with particular sets of characteristics unique to their species

(C) Some species become extinct because of accumulated gradual changes in their local environments

(D) In geologically recent times for which there is no fossil record human intervention has changed the pattern of extinctions

(E) Species that are widely dispersed are the least likely to become extinct

10. Which of the following best completes the passage below?

At a recent conference on environmental threats to the North Seamost participating countries favored uniform controls on the quality of effluents whether or not specific environmental damage could be attributed to a particular source of effluent What must of course be shown in order to avoid excessively restrictive controls is that  .

(A) any uniform controls that are adopted are likely to be implemented without delay

(B) any substance to be made subject to controls can actually cause environmental damage

(C) the countries favoring uniform controls are those generating the largest quantities of effluents

(D) all of any given pollutant that is to be controlled actually reaches the North Sea at present

(E) environmental damage already inflicted on the North Sea is reversible

11. Shelby Industries manufactures and sells the same gauges as Jones Industries Employee wages account for forty percent of the cost of manufacturing gauges at both Shelby Industries and Jones Industries Shelby Industries is seeking a competitive advantage over Jones Industries Therefore to promote this end Shelby Industries should lower employee wages

Which of the following if true would most weaken the argument above?

(A) Because they make a small number of precision instruments gauge manufacturers cannot receive volume discounts on raw materials

(B) Lowering wages would reduce the quality of employee work and this reduced quality would lead to lowered sales

(C) Jones Industries has taken away twenty percent of Shelby Industries business over the last year.

(D) Shelby Industries pays its employees on average ten percent more than does Jones Industries

(E) Many people who work for manufacturing plants live in areas in which the manufacturing plant they work for is the only industry

12. Large national budget deficits do not cause large trade deficits If they did countries with the largest budget deficits would also have the largest trade deficits In fact when deficit figures are adjusted so that different countries are reliably comparable to each other, there is no such correlation

If the statements above are all true, which of the following can properly be inferred on the basis of them?

(A) Countries with large national budget deficits tend to restrict foreign trade

(B) Reliable comparisons of the deficit figures of one country with those of another are impossible

(C) Reducing a country's national budget deficit will not necessarily result in a lowering of any trade deficit that country may have

(D) When countries are ordered from largest to smallest in terms of population the smallest countries generally have the smallest budget and trade deficits

(E) Countries with the largest trade deficits never have similarly large national budget deficits

A famous singer recently won a lawsuit against an advertising firm for using another singer in a commercial to evoke the famous singers well-known rendition of a certain song As a result of the lawsuit advertising firms will stop using imitators in commercials Therefore advertising costs will rise since famous singers services cost more than those of their imitators

The conclusion above is based on which of the following assumptions?

(A) Most people are unable to distinguish a famous singer's rendition of a song from a good imitator's rendition of the same song

(B) Commercials using famous singers are usually more effective than commercials using imitators of famous singers

(C) The original versions of some well known songs are unavailable for use in commercials

(D) Advertising firms will continue to use imitators to mimic the physical mannerisms of famous singers

The advertising industry will use well-known renditions of songs in commercials

The sustained massive use of pesticides in farming has two effects that are especially pernicious First, it often kills off the pests natural enemies in the area Second it often unintentionally gives rise to insecticide-resistant pests since those insects that survive a particular insecticide will be the ones most resistant to it and they are the ones left to breed

From the passage above it can be properly inferred that the effectiveness of the sustained massive use of pesticides can be extended by doing which of the following assuming that each is a realistic possibility?

(A) Using only chemically stable insecticides

(B) Periodically switching the type of insecticide used

(C) Gradually increasing the quantities of pesticides used

(D) Leaving a few fields fallow every year

(E) Breeding higher-yielding varieties of crop plants

Treatment for hypertension forestalls certain medical expenses by preventing strokes and heart disease Yet any money so saved amounts to only onefourth of the expenditures required to treat the hypertensive population Therefore there is no economic 1ustification for preventive treatment for hypertension

Which of the following if true is most damaging to the conclusion above?

(A) The many fatal strokes and heart attacks resulting from untreated hypertension cause insignificant medical expenditures but large economic losses of other sorts

(B) The cost per patient of preventive treatment for hypertension would remain constant even if such treatment were instituted on a large scale

(C) In matters of health care Economic considerations should ideally not be dominant

(D) Effective prevention presupposes early diagnosis and programs to ensure early diagnosis are costly

(E) The net savings in medical resources achieved by some preventive health measures are smaller than the net losses attributable to certain other measures of this kind

16. In an attempt to promote the widespread use of paper rather than plastic and thus reduce non-biodegradable waste the council of a small town plans to ban the sale of disposable plastic goods for which substitutes made of paper exist The council argues that since most paper is entirely biodegradable paper goods are environmentally preferable

Which of the following if true indicates that the plan to ban the sale of disposable plastic goods is suited to the town council's environmental goals?

(A) Although biodegradable plastic goods are now available members of the town council believe biodegradable paper goods to be safer for the environment

(B) The paper factory at which most of the townspeople are employed plans to increase production of biodegradable paper goods

(C) After other towns enacted similar bans on the sale of plastic goods the environmental benefits were not discernible for several years

(D) Since most townspeople prefer plastic goods to paper goods in many instances they are likely to purchase them in neighboring towns where plastic goods are available for sale.

(E) Products other than those derived from wood pulp are often used in the manufacture of paper goods that are entirely biodegradable

17. Since the deregulation of airlines delays at the nation's increasingly busy airports have increased by 25 percent To combat this problem more of the takeoff and landing slots at the busiest airports must be allocated to commercial airlines

Which of the following if true casts the most doubt on the effectiveness of the solution proposed above?

(A) The major causes of delays at the nation's busiest airports are bad weather and overtaxed air traffic control equipment

(B) Since airline deregulation began the number of airplanes in operation has increased by 25 percent

(C) Over 60 percent of the takeoff and landing slots at the nation's busiest airports are reserved for commercial airlines

(D) After a small Midwestern airport doubled its allocation of takeoff and landing slots the number of delays that were reported decreased by 50 percent

(E) Since deregulation the average length of delay at the nation's busiest airports has doubled

18. Unlike the wholesale price of raw wool the wholesale price of raw cotton has fallen considerably in the last yean Thus although the retail price of cotton clothing at retail clothing stores has not yet fallen it will inevitably fall

Which of the following if true most seriously weakens the argument above?

(A) The cost of processing raw cotton for cloth has increased during the last year.

(B) The wholesale price of raw wool is typically higher than that of the same volume of raw cotton

(C) The operating costs of the average retail clothing store have remained constant during the last year.

(D) Changes in retail prices always lag behind changes in wholesale prices

(E) The cost of harvesting raw cotton has increased in the last year

19. Reviewer The book Arts Decline argues that European painters today lack skills that were common among European painters of preceding centuries In this the book must be right, since its analysis of 100 paintings 50 old and 50 contemporary, demonstrates convincingly that none of the contemporary paintings are executed as skillfully as the older paintings

Which of the following points to the most serious logical flaw in the reviewer's argument?

(A) The paintings chosen by the book's author for analysis could be those that most support the book's thesis

(B) There could be criteria other than the technical skill of the artist by which to evaluate a painting

(C) The title of the book could cause readers to accept the book's thesis even before they read the analysis of the paintings that supports it

(D) The particular methods currently used by European painters could require less artistic skill than do methods used by painters in other parts of the world

(E) A reader who was not familiar with the language of art criticism might not be convinced by the book's analysis of the 100 paintings

20.A computer equipped with signature-recognition software, which restricts access to a computer to those people whose signatures are on file identifies a person's signature by analyzing not only the form of the signature but also such characteristics as pen pressure and signing speed Even the most adept forgers cannot duplicate all of the characteristics the program analyzes

Which of the following can be logically concluded from the passage above?

(A) The time it takes to record and analyze a signature makes the software impractical for everyday use

(B) Computers equipped with the software will soon be installed In most banks

(C) Nobody can gain access to a computer equipped with the software solely by virtue of skill at forging signatures

(D) Signature-recognition software has taken many years to develop and perfect

(E) In many cases even authorized users are denied legitimate access to computers equipped with the software

21. Startup companies financed by venture capitalists have a much lower failure rate than companies financed by other means Source of financing therefore must be a more important causative factor in the success of a start-up company than are such factors as the personal characteristics of the entrepreneur, the quality of strategic planning or the management structure of the company

Which of the following if true most seriously weakens the argument above?

(A) Venture capitalists tend to be more responsive than other sources of financing to changes in a start-up company's financial needs

(B) The strategic planning of a start-up company is a less important factor in the long-term success of the company than are the personal characteristics of the entrepreneur

(C) More than half of all new companies fail within five years

(D) The management structures of start-up companies are generally less formal than the management structures of ongoing businesses

(E) Venture capitalists base their decisions to fund start up companies on such factors as the characteristics of the entrepreneur and quality of strategic planning of the company.

Questions 22-23 are based on the following

Half of the subjects in an experiment-the experimental group--consumed large quantities of a popular artificial sweetener Afterward this group showed lower cognitive abilities than did the other half of the subjects-the control group who did not consume the sweetener The detrimental effects were attributed to an amino acid that is one of the sweetener's principal constituents

Which of the following if true would best support the conclusion that some ingredient of the sweetener was responsible for the experimental results?

(A) Most consumers of the sweetener do not consume as much of it as the experimental group members did

(B) The amino acid referred to in the conclusion is a component of all proteins some of which must be consumed for adequate nutrition

(C) The quantity of the sweetener consumed by individuals in the experimental group is considered safe by federal food regulators

(D) The two groups of subjects were evenly matched with regard to cognitive abilities prior to the experiment

(E) A second experiment in which subjects consumed large quantities of the sweetener lacked a control group of subjects who were not given the sweetener,

Which of the following if true would best help explain how the sweetener might produce the observed effect?

(A) The government's analysis of the artificial sweetener determined that it was sold in relatively pure form

(B) A high level of the amino acid In the blood inhibits the synthesis of a substance required for normal brain functioning

(C) Because the sweetener is used primarily as a food additive adverse reactions to it are rarely noticed by consumers

(D) The amino acid that is a constituent of the sweetener is also sold separately as a dietary supplement

(E) Subjects in the experiment did not know whether they were consuming the sweetener or a second harmless substance

2 In the arid land along the Colorado River, use of the river's water supply is strictly controlled farms along the river each have a limited allocation that they are allowed to use for irrigation But the trees that grow in narrow strips along the rivers banks also use its water Clearly, therefore if farmers were to remove those trees more water would be available for crop irrigation

Which of the following if true most seriously weakens the argument?

(A) The trees along the river's banks shelter it from the sun and wind thereby greatly reducing the amount of water lost through evaporation

(B) Owners of farms along the river will probably not undertake the expense of cutting down trees along the banks unless they are granted a greater allocation of water in return

(C) Many of the tree species currently found along the river's banks are specifically adapted to growing in places where tree roots remain constantly wet

(D) The strip of land where trees grow along the river's banks would not be suitable for growing crops if the trees were removed

(E) The distribution of water allocations for irrigation is intended to prevent farms farther upstream from using water needed by farms farther downstream

Near Chicago a newly built hydroponic spinach factory, a completely controlled environment for growing spinach produces on 1 acre of f1oor space what it takes 100 acres of fields to produce Expenses especially for electricity, are high however, and the spinach produced costs about four times as much as washed California field spinach the spinach commonly sold throughout the United States

Which of the following if true best supports a projection that the spinachgrowing facility near Chicago will be profitable?

(A) Once the operators of the facility are experienced they will be able to cut operating expenses by about 25 percent

(B) There is virtually no scope for any further reduction in the cost per pound for California field spinach

(C) Unlike washed field spinach the hydroponically grown spinach is untainted by any pesticides or herbicides and thus will sell at exceptionally high prices to such customers as health food restaurants

(D) Since spinach is a crop that ships relatively well the market for the hydroponically grown spinach is no more limited to the Chicago area than the market for California field spinach is to California

(E) A second hydroponic facility is being built in Canada taking advantage of inexpensive electricity and high vegetable prices

Automobile Dealer's Advertisement

The Highway Traffic Safety Institute reports that the PZ 1000 has the fewest injuries per accident of any car in its class This shows that the PZ 1000 is one of the safest cars available today

Which of the following if true most seriously weakens the argument in the advertisement?

(A) The Highway Traffic Safety Institute report listed many cars in other classes that had more injuries per accident than did the PZ 1000

(B) In recent years many more PZ 1000's have been sold than have any other kind of car in its class

(C) Cars in the class to which the PZ 1000 belongs are more likely to be involved in accidents than are other types of cars

(D) The difference between the number of injuries per accident for the PZ 1000 and that for other cars in its class is quite pronounced

(E) The Highway Traffic Safety Institute issues reports only once a yean

27. Editorial The mayor plans to deactivate the citys fire alarm boxes because most calls received from them are false alarms The mayor claims that the alarm boxes are no longer necessary, since most people now have access to either public or private telephones But the citys commercial district where there is the greatest risk of fire has few residents and few public telephones so some alarm boxes are still necessary

Which of the following if true most seriously weakens the editorial's argument?

(A) Maintaining the fire alarm boxes costs the city more than five million dollars annually

(B) Commercial buildings have automatic fire alarm systems that are linked directly to the fire department

(C) The fire department gets less information from an alarm box than it does from a telephone call

(D) The city s fire department is located much closer to the residential areas than to the commercial district

(E) On average almost 25 percent of the public telephones in the city are out of order

State spokesperson Many businesspeople who have not been to our state believe that we have an inadequate road system Those people are mistaken as is obvious from the fact that in each of the past six years our state has spent more money per mile on road improvements than any other state

Which of the following if true most seriously undermines the reasoning in the spokespersons argument?

(A) In the spokesperson's state spending on road improvements has been increasing more slowly over the past six years than it has in several other states

(B) Adequacy of a state's road system is generally less important to a businessperson considering doing business there than is the availability of qualified employees

(C) Over the past six years numerous businesses have left the spokesperson's state but about as many businesses have moved into the state.

(D) In general the number of miles of road in a state's road system depends on both the area and the population of the state

(E) Only states with seriously inadequate road systems need to spend large amounts of money on road improvements

29. A program instituted in a particular state allows parents to prepay their children's future college tuition at current rates The program then pays the tuition annually for the child at any of the states public colleges in which the child enrolls Parents should participate in the program as a means of decreasing the cost for their children's college education

Which of the following if true is the most appropriate reason for parents not to participate in the program?

(A) The parents are unsure about which public college in the state the child will attend

(B) The amount of money accumulated by putting the prepayment funds in an interest-bearing account today will be greater than the total cost of tuition for any of the public colleges when the child enrolls

(C) The annual cost of tuition at the state's public colleges is expected to increase at a faster rate than the annual increase in the cost of living

(D) Some of the state's public colleges are contemplating large increases in tuition next year

(E) The prepayment plan would not cover the cost of room and board at any of the state's public colleges

Company Alpha buys free-travel coupons from people who are awarded the coupons by Bravo Airlines for flying frequently on Bravo airplanes The coupons are sold to people who pay less for the coupons than they would pay by purchasing tickets from Bravo This marketing of coupons results In lost revenue for Bravo

To discourage the buying and selling of free travel coupons it would be best for Bravo Airlines to restrict the

(A) number of coupons that a person can be awarded in a particular year

(B) use of the coupons to those who were awarded the coupons and members of their immediate families

(C) days that the coupons can be used to Monday through Friday

(D) amount of time that the coupons can be used after they are issued

(E) number of routes on which travelers can use the coupons

Toughened hiring standards have not been the primary cause of the present staffing shortage in public schools The shortage of teachers is primarily caused by the fact that in recent years teachers have not experienced any improvements in working conditions and their salaries have not kept pace with salaries in other professions

Which of the following if true would most support the claims above?

(A) Many teachers already In the profession would not have been hired under the new hiring standards

(B) Today more teachers are entering the profession with a higher educational level than in the past

(C) Some teachers have cited higher standards for hiring as a reason for the current staffing shortage

(D) Many teachers have cited low pay and lack of professional freedom as reasons for their leaving the profession

(E) Many prospective teachers have cited the new hiring standards as a reason for not entering the profession

A proposed ordinance requires the installation in new homes of sprinklers automatically triggered by the presence of a fire However, a home builder argued that because more than 90 percent of residential fires are extinguished by a household member, residential sprinklers would only marginally decrease property damage caused by residential fires

Which of the following if true would most seriously weaken the home builder's argument?

(A) Most individuals have no formal training in how to extinguish fires

(B) Since new homes are only a tiny percentage of available housing in the city, the new ordinance would be extremely narrow In scope

(C) The installation of smoke detectors in new residences costs significantly less than the installation of sprinklers

(D) In the city where the ordinance was proposed the average time required by the fire department to respond to a fire was less than the national average

(E) The largest proportion of property damage that results from residential fires is caused by fires that start when no household member is present

A recent spate of launching and operating mishaps with television satellites led to a corresponding surge in claims against companies underwriting satellite insurance As a result, insurance premiums shot up making satellites more expensive to launch and operate This in turn had added to the pressure to squeeze more performance out of currently operating satellites

Which of the following if true taken together with the information above best supports the conclusion that the cost of television satellites will continue to increase?

(A) Since the risk to insurers of satellites is spread over relatively few units insurance premiums are necessarily very high

(B) When satellites reach orbit and then fail, the causes of failure are generally impossible to pinpoint with confidence

(C) The greater the performance demands placed on satellites the more frequently those satellites break down

(D) Most satellites are produced in such small numbers that no economies of scale can be realized

(E) Since many satellites are built by unwieldy international consortia, inefficiencies are inevitable

Questions 34-35 refer to the following

If the airspace around centrally located airports were restricted to commercial airliners and only those private planes equipped with radar, most of the private-plane traffic would be forced to use outlying airfields Such a reduction in the amount of private-plane traffic would reduce the risk of midair collision around the centrally located airports

The conclusion drawn in the first sentence depends on which of the following assumptions?

(A) Outlying airfields would be as convenient as centrally located airports for most pilots of private planes

(B) Most outlying airfields are not equipped to handle commercial-airline traffic

(C) Most private planes that use centrally located airports are not equipped with radar

(D) Commercial airliners are at greater risk of becoming involved in midair collisions than are private planes

(E) A reduction in the risk of midair collision would eventually lead to increases in commercial airline traffic

Which of the following, if true, would most strengthen the conclusion drawn in the second sentence?

(A) Commercial airliners are already required by law to be equipped with extremely sophisticated radar systems

(B) Centrally located airports are experiencing overcrowded airspace primarily because of sharp increases in commercial-airline traffic

(C) Many pilots of private planes would rather buy radar equipment than be excluded from centrally located airports.

(D) The number of midair collisions that occur near centrally located airports has decreased in recent years

(E) Private planes not equipped with radar systems cause a disproportionately large number of midair collisions around centrally located airports

Two decades after the Emerald River Dam was built, none of the eight fish species native to the Emerald River was still reproducing adequately in the river below the dam Since the dam reduced the annual range of water temperature in the river below the dam from50 degrees to 6 degrees, scientists have hypothesized that sharply rising water temperatures must be involved in signaling the native species to begin the reproductive cycle

Which of the following statements if true would most strengthen the scientists hypothesis?

(A) The native fish species were still able to reproduce only in side streams of the river below the dam where the annual temperature range remains approximately 50degrees

(B) Before the dam was built the Emerald River annually overflowed its banks creating backwaters that were critical breeding areas for the native species of fish

(C) The lowest recorded temperature of the Emerald River before the dam was built was 34 degrees whereas the lowest recorded temperature of the river after the dam was built has been 43 degrees

(D) Nonnative species of fish introduced into the Emerald River after the dam was built, have begun competing with the declining native fish species for food and space.

(E) Five of the fish species native to the Emerald River are not native to any other river in North America

37. Certain messenger molecules fight damage to the lungs from noxious air by telling the muscle cells encircling the lungs airways to contract This partially seals off the lungs An asthma attack occurs when the messenger molecules are activated unnecessarily, in response to harmless things like pollen or household dust

Which of the following if true points to the most serious flaw of a plan to develop a medication that would prevent asthma attacks by blocking receipt of any messages sent by the messenger molecules referred to above?

(A) Researchers do not yet know how the body produces the messenger molecules that trigger asthma attacks.

(B) Researchers do not yet know what makes one person's messenger molecules more easily activated than another's

(C) Such a medication would not become available for several years because of long lead times in both development and manufacture

(D) Such a medication would be unable to distinguish between messages triggered by pollen and household dust and messages triggered by noxious air

(E) Such a medication would be a preventative only and would be unable to alleviate an asthma attack once it had started

38. Which of the following best completes the passage below?

The more worried investors are about losing their money, the more they will demand a high potential return on their investment great risks must be offset by the chance of great rewards This principle is the fundamental one in determining interest rates and it is illustrated by the fact that 

(A) successful investors are distinguished by an ability to make very risky investments without worrying about their money

(B) lenders receive higher Interest rates on unsecured loans than on loans backed by collateral

(C )in times of high inflation the interest paid to depositors by banks can actually be below the rate of inflation

(D) at any one time a commercial bank will have a single rate of interest that it will expect all of its individual borrowers to pay

(E) the potential return on investment in a new company is typically lower than the potential return on investment in a well-established company

39. A certain mayor has proposed a fee of five dollars per day on private vehicles entering the city, claiming that the fee will alleviate the citys traffic congestion The mayor reasons that since the fee will exceed the cost of round-trip bus fare from many nearby points many people will switch from using their cars to using the bus Which of the following statements, if true, provides the best evidence that the mayor's reasoning is flawed?

(A) Projected increases in the price of gasoline will Increase the cost of taking a private vehicle into the city

(B) The cost of parking fees already makes it considerably more expensive for most people to take a private vehicle into the city than to take a bus

(C) Most of the people currently riding the bus do not own private vehicles

(D) Many commuters opposing the mayors plan have indicated that they would rather endure traffic congestion than pay a five-dollar-per-day fee

(E) During the average workday, private vehicles owned and operated by people living within the city account for 20 percent of the city's traffic congestion

40. Dental researchers recently discovered that tooth-brushes can become contaminated with bacteria that cause pneumonia and strep throat They found that contamination usually Occurs after toothbrushes have been used for four weeks For that reason people should replace their toothbrushes at least once a month

Which of the following if true would most weaken the conclusion above?

(A) The dental researchers could not discover why toothbrush contamination usually occurred only after toothbrushes had been used for four weeks

(B) The dental researchers failed to investigate contamination of toothbrushes by viruses yeasts and other pathogenic microorganisms

(C) The dental researchers found that among people who used toothbrushes contaminated with bacteria that cause pneumonia and strep throat the incidence of these diseases was no higher than among people who used uncontaminated toothbrushes

(D) The dental researchers found that people who rinsed their toothbrushes thoroughly in hot water after each use were as likely to have contaminated toothbrushes as were people who only rinsed their toothbrushes hurriedly in cold water after each use

(E) The dental researchers found that, after six weeks of use greater length of use of a toothbrush did not correlate with a higher number of bacteria being present

Leaders of a miners union on strike against Coalco are contemplating additional measures to pressure the company to accept the union's contract proposal. The union leaders are considering as their principal new tactic a consumer boycott against Gasco gas stations which are owned by Energy Incorporated the same corporation that owns Coaico

The answer to which of the following questions is LEAST directly relevant to the union leaders consideration of whether attempting a boycott of Gasco will lead to acceptance of their contract proposal?

(A) Would revenue losses by Gasco seriously affect Energy Incorporated?

(B) Can current Gasco customers easily obtain gasoline elsewhere?

(C) Have other miners unions won contracts similar to the one proposed by this union?

(D) Have other unions that have employed a similar tactic achieved their goals with it?

(E) Do other corporations that own coal companies also own gas stations?

Questions 42-43 are based on the following

Transnational cooperation among corporations is experiencing a modest renaissance among United States firms even though projects undertaken by two or more corporations under a collaborative agreement are less profitable than projects undertaken by a single corporation The advantage of transnational cooperation is that such joint international projects may allow United States firms to win foreign contracts that they would not otherwise be able to win

42. Which of the following statements by a United States corporate officer best fits the situation of United States firms as described in the passage above?

(A) We would rather make only a share of the profit and also risk only a share of a possible loss than run the full risk of a loss

(B) We would rather make a share of a relatively modest profit than end up making none of a potentially much bigger profit

(C) We would rather cooperate and build good will than poison the business climate by all-out competition

(D) We would rather have foreign corporations join us in American projects than join them in projects in their home countries

(E) We would rather win a contract with a truly competitive bid of our own than get involved in less profitable collaborative agreements

Which of the following is information provided by the passage above?

(A) Transnational cooperation involves projects too big for a single corporation to handle

(B) Transnational cooperation results in a pooling of resources leading to high-quality performance

(C) Transnational cooperation has in the past been both more common and less common than it Is now among United States firms

(D) Joint projects between United States and foreign corporations are not profitable enough to be worth undertaking

(E) Joint projects between United States and foreign corporations benefit only those who commission the projects

4 Laws requiring the use of headlights during daylight hours can prevent automobile collisions However, since daylight visibility is worse in countries farther from the equator, any such laws would obviously be more effective in preventing collisions in those countries In fact the only countries that actually have such laws are farther from the equator than is the continental United States

Which of the following conclusions could be most properly drawn from the information given above?

(A) Drivers in the continental United States who used their headlights during the day would be just as likely to become involved in a collision as would drivers who did not use their headlights

(B) In many countries that are farther from the equator than is the continental United States poor daylight visibility is the single most important factor in automobile collisions

(C) The proportion of automobile collisions that occur in the daytime is greater in the continental United States than in the countries that have daytime headlight laws

(D) Fewer automobile collisions probably occur each year in countries that have daytime headlight laws than occur within the continental United States

(E) Daytime headlight laws would probably do less to prevent automobile collisions in the continental United States than they do in the countries that have the laws.

Questions 45_46 al e based on the following

Bank depositors in the United States are all financially protected against bank failure because the government insures all individuals bank deposits An economist argues that this insurance is partly responsible for the high rate of bank failures since it removes from depositors any financial incentive to find out whether the bank that holds their money is secure against failure If depositors were more selective then banks would need to be secure in order to compete for dositors money

The economist's argument makes which of the following assumptions?

(A) Bank failures are caused when big borrowers default on loan repayments

(B) A significant proportion of depositors maintain accounts at several different banks

(C) The more a deD0sitor has to deposit, the more careful he or she tends to be in selecting a bank

(D) The difference in the interest rates paid to depositors by different banks is not a significant factor in bank failures

(E) Potential depositors are able to determine which banks are secure against failure

Which of he following if true most seriously weakens the economist s argument?

(A) Before the government started to insure depositors against bank failure there was a lower rate of bank failure than there is now

(B) When the government did not insure deposits frequent bank failures occurred as a result of depositors fears of losing money in bank failures

(C) Surveys show that a significant proportion of depositors are aware that their deposits are insured by the government

(D) There is an upper limit on the amount of an individual's deposit that the government will insure but very few individuals deposits exceed this limit

(E) The security of a bank against failure depends on the percentage of its assets that are loaned out and also on how much risk its loans involve

A study of marital relationships in which one partner's sleeping and waking cycles differ from those of the other partner reveals that such couples share fewer activities with each other and have more violent arguments than do couples in a relationship in which both partners follow the same sleeping and waking patterns. Thus mismatched sleeping and waking cycles can seriously jeopardize a marriage

Which of the following if true most seriously weakens the argument above?

(A) Married couples in which both spouses follow the same sleeping and waking patterns also occasionally have arguments that can jeopardize the couple's marriage

(B) The sleeping and waking cycles of individuals tend to vary from season to season

(C) The Individuals who have sleeping and waking cycles that differ significantly from those of their spouses tend to argue little with colleagues at work

(D) People in unhappy marriages have been found to express hostility by adopting different sleeping and waking cycles from those of their spouses

(E) According to a recent study, most people's sleeping and waking cycles can be controlled and modified easily

48. In the past most airline companies minimized aircraft weight to minimize fuel costs The safest airline seats were heavy, and airlines equipped their planes with few of these seats This year the seat that has sold best to airlines has been the safest Onea clear indication that airlines are assigning a higher priority to safe seating than to minimizing fuel costs

Which of the following if true most seriously weakens the argument above?

(A) Last year's best-selling airline seat was not the safest airline seat 0n the market

(B) No airline company has announced that it would be making safe seating a higher priority this year

(C) The price of fuel was higher this year than it had been in most of the years when the safest airline seats sold poorly

(D) Because of increases in the cost of materials all airline seats were more expensive to manufacture this year than in any previous year

(E) Because of technological innovations the safest airline seat on the market this year weighed less than most other airline seats on the market

Division Manager I want to replace the Microton computers in my division with Vitech computers

General Manager Why?

Division Manager It costs 28 percent less to train new staff on the Vitech

General Manager But that is not a good enough reason We can simply hire only people who already know how to use the Microton computer

Which of the following if true most seriously undermines the general manager's objection to the replacement of Microton computers with Vitechs?

(A) Currently all employees in the company are required to attend workshops on how to use Microton computers in new applications

(B) Once employees learn how to use a compute, they tend to change employers more readily than before

(C) Experienced users of Microton computers command much higher salaries than do prospective employees who have no experience in the use of computers

(D) The average productivity of employees in the general managers company is below the average productivity of the employees of its competitors

(E) The high costs of replacement parts make Vitech computers more expensive to maintain than Microton computers

50. Crops can be traded on the futures market before they are harvested If a poor corn harvest is predicted prices of corn futures rise if a bountiful corn harvest is predicted prices of corn futures fall This morning meteorologists are predicting much-needed rain for the corn-growing region starting tomorrow Therefore since adequate moisture is essential for the current crop's survival prices of corn futures will fall sharply today

Which of the following if true most weakens the argument above?

(A) Corn that does not receive adequate moisture during its critical pollination stage will not produce a bountiful harvest

(B) Futures prices for corn have been fluctuating more dramatically this season than last season.

(C) The rain that meteorologists predicted for tomorrow is expected to extend well beyond the corn-growing region

(D) Agriculture experts announced today that a disease that has devastated some of the corn crop will spread widely before the end of the growing season

(E) Most people who trade in Com futures rarely take physical possession of the corn they trade

The interview is an essential part of a successful hiring program because with it job applicants who have personalities that are unsuited to the requirements of the job will be eliminated from consideration

The argument above logically depends on which of the following assumptions?

(A) A hiring program will be successful if it includes interviews

(B) The interview is a more important part of a successful hiring program than is the development of a job description

(C) Interviewers can accurately identify applicants whose personalities are unsuited to the requirements of the job

(D) The only purpose of an interview is to evaluate whether job applicants personalities are suited to the requirements of the job

(E) The fit of job applicants personalities to the requirements of the job was once the most Important factor in making hiring decisions

Useful protein drugs such as insulin must still be administered by the cumbersome procedure of injection under the skin If proteins are taken orally, they are digested and cannot reach their target cells Certain no protein drugs however, contain chemical bonds that are not broken down by the digestive system They can thus betaken orally

The statements above most strongly support a claim that a research procedure that successfully accomplishes which of the following would be beneficial to users of protein drugs?

(A) Coating insulin with compounds that are broken down by target cells but whose chemical bonds are resistant to digestion

(B) Converting into protein compounds by procedures that work in the laboratory, the no protein drugs that resist digestion

(C) Removing permanently from the digestive system any substances that digest proteins

(D) Determining in a systematic way, what enzymes and bacteria are present in the normal digestive system and whether they tend to be broken down within the body

(E) Determining the amount of time each no protein drug takes to reach its target cells

There is a great deal of geographical variation in the frequency of many surgical proceduresup to tenfold variation per hundred thousand people among different areas in the numbers of hysterectomies, prostatectomies, and tonsillectomies

To support a conclusion that much of the variation is due to unnecessary surgical procedures it would be most important to establish which of the following?

(A) A local board of review at each hospital examines the records of every operation to determine whether the surgical procedure was necessary

(B) The variation is unrelated to factors (other than the surgical procedures themselves) that influence the incidence of diseases for which surgery might be considered

(C) There are several categories of surgical procedure(other than hysterectomies prostatectomies and tonsillectomies)that are often performed unnecessarily

(D) For certain surgical procedures it is difficult to determine after the operation whether the procedures were necessary or whether alternative treatment would have succeeded

(E) With respect to how often they are performed unnecessarily, hysterectomies prostatectomies and tonsillectomies are representative of surgical procedures in general

5 A museum has been offered an undocumented statue supposedly Greek and from the sixth century B C Possibly the statue is genuine but undocumented because it was recently unearthed or because it has been privately owned However, an ancient surface usually has uneven weathering whereas the surface of this statue has the uniform quality characteristically produced by a chemical bath used by forgers to imitate a weathered surface Therefore the statue is probably a forgery.

Which of the following if true most seriously weakens the argument?

(A) Museums can accept a recently unearthed statue only with valid export documentation from its country of origin

(B) The subject's pose and other aspects of the subject's treatment exhibit all the most common features of Greek statues of the sixth century B C

(C) The chemical bath that forgers use was at one time used by dealers and collectors to remove the splotchy surface appearance of genuinely ancient sculptures

(D) Museum officials believe that forgers have no technique that can convincingly simulate the patchy weathering characteristic of the surfaces of ancient sculptures

(E) An allegedly Roman sculpture with a uniform surface Similar to that of the statue being offered to the museum was recently shown to be a forgery

55. Studies have shown that elderly people who practice a religion are much more likely to die immediately after an important religious holiday period than immediately before one Researchers have concluded that the will to live can prolong life at least for short periods of time

Which of the following if true would most strengthen the researchers conclusion?

(A) Elderly people who practice a religion are less likely to die immediately before or during an important religious holiday than at any other time of the yean

(B) Elderly people who practice a religion appear to experience less anxiety at the prospect of dying than do other people

(C) Some elderly people who do practice a religion live much longer than most elderly people who do not

(D) Most elderly people who participate in religious holidays have different reasons for participating than young people do

(E) Many religions have important holidays in the spring and fall seasons with the lowest death rates for elderly people

56. Gortland has long been narrowly self-sufficient in both grain and meat However, as per capita income in Gortland has risen toward the world average per capita consumption of meat has also risen toward the world average and it takes several pounds of grain to produce one pound of meat. Therefore since per capita income continues to rise whereas domestic grain production will not increase Gortland will soon have to import either grain or meat or both

Which of the following is an assumption on which the argument depends?

(A) The total acreage devoted to grain production in Gortland will not decrease substantially

(B) The population of Gortland has remained relatively constant during the country's years of growing prosperity.

(C) The per capita consumption of meat in Gortland is roughly the same across all income levels

(D) In Gortland, neither meat nor grain is subject to government price controls

(E) People in Gortland who increase their consumption of meat will not radically decrease their consumption of grain

57. Meteorite explosions in the Earth's atmosphere as large as the one that destroyed forests in Siberia with approximately the force of a 12-megaton nuclear blast occur about once a century

The response of highly automated systems controlled by complex computer programs to unexpected circumstances is unpredictable

Which of the following conclusions can most properly be drawn if the statements above are true about a highly automated nuclear-missile defense system controlled by a complex computer program?

(A) Within a century after its construction the system would react inappropriately and might accidentally start a nuclear war.

(B) The system would be destroyed if an explosion of a large meteorite occurred in the Earth's atmosphere

(C) It would be impossible for the system to distinguish the explosion of a large meteorite from the explosion of a nuclear weapon

(D) Whether the system would respond inappropriately to the explosion of a large meteorite would depend on the location of the blast

(E) It is not certain what the system's response to the explosion of a large meteorite would be, if its designers did not plan for such a contingency

Questions 58-59 are based on the following

If there is an oil-supply disruption resulting in higher international oil prices domestic oil prices in open-market counties such as the United States will rise as well whether such countries import all or none of their oil

58. If the statement above concerning oil-supply disruptions is true which of the following policies in an open-market nation is most likely to reduce the long-term economic impact on that nation of sharp and unexpected increases in international oil prices?

(A) Maintaining the quantity of oil imported at constant yearly levels

(B) Increasing the number of oil tankers in its fleet

(C) Suspending diplomatic relations with major oil-producing nations

(D) Decreasing oil consumption through conservation

(E) Decreasing domestic production of oil

Which of the following conclusions is best supported by the statement above?

(A) Domestic producers of oil in open-market countries are excluded from the international oil market when there is a disruption in the international oil supply

(B) International oil-supply disruptions have little if any, effect on the price of domestic oil as long as an open-market country has domestic supplies capable of meeting domestic demand

(C) The oil market in an open-market country is actually part of the international oil market even-f most of that country's domestic oil is usually sold to consumers within its borders

(D) Open-market countries that export little or none of their oil can maintain stable domestic oil prices even when international oil prices rise sharply

(E) If international oil prices rise domestic distributors of oil in open-market countries will begin to import more oil than they export

60. Most archaeologists have held that people first reached the Americas less than 20 000 years ago by crossing a land bridge into North America But recent discoveries of human shelters in South America dating from 32 000 years ago have led researchers to speculate that people arrived in South America first, after voyaging across the Pacific and then spread northward

Which of the following if it were discovered would be pertinent evidence against the speculation above?

(A) A rock shelter near Pittsburgh Pennsylvania contains evidence of use by human beings 19,000 years ago.

(B) Some North American sites of human habitation predate any sites found in South America

(C) The climate is warmer at the 32,000-year-old South American site than at the oldest known North American site

(D) The site in South America that was occupied 32,000 years ago was continuously occupied until 6,000 years ago

(E) The last Ice Age between 11 500 and 20,000 years ago considerably lowered worldwide sea levels

61.The tobacco industry is still profitable and projections are that it will remain so In the United States this year, the total amount of tobacco sold by tobacco farmers has increased even though the number of adults who smoke has decreased

Each of the following if true could explain the simultaneous increase in tobacco sales and decrease in the number of adults who smoke EXCEPT

(A) During this year, the number of women who have begun to smoke is greater than the number of men who have quit smoking.

(B) The number of teenage children who have begun to smoke this year is greater than the number of adults who have quit smoking during the same period

(C) During this yea5 the number of nonsmokers who have begun to use chewing tobacco or snuff is greater than the number of people who have quit smoking

(D) The people who have continued to smoke consume more tobacco per person than they did in the past

(E) More of the cigarettes made in the United States this year were exported to other countries than was the case last year.

A milepost on the towpath read on the side facing the hiker as she approached it and on its back She reasoned that the next milepost forward on the path would indicate that she was halfway between one end of the path and the other. However, the milepost one mile further on read facing her and behind

Which of the following if true would explain the discrepancy described above?

(A) The numbers on the next milepost had been reversed

(B) The numbers on the mileposts indicate kilometers not miles

(C) The facing numbers indicate miles to the end of the path not miles from the beginning

(D) A milepost was missing between the two the hiker encountered

(E) The mileposts had originally been put in place for the use of mountain bikers not for hikers

63. Traditionally, decision making by managers that is reasoned step-by-step has been considered preferable to intuitive decision making However, a recent study found that top managers used intuition significantly more than did most middle-or lower level managers This confirms the alternative view that intuition is actually more effective than careful methodical reasoning

The conclusion above is based on which of the following assumptions?

(A) Methodical step-by-step reasoning is inappropriate for making many real-life management decisions

(B) Top managers have the ability to use either Intuitive reasoning or methodical step-by-step reasoning in making decisions

(C) The decisions made by middle-and lower-level managers can be made as easily by using methodical reasoning as by using intuitive reasoning

(D) Top managers use intuitive reasoning in making the majority of their decisions

(E) Top managers are more effective at decision making than middle-or lower-level managers

6 High levels of fertilizer and pesticides needed when farmers try to produce high yields of the same crop year after yea5 pollute water supplies Experts therefore urge farmers to diversify their crops and to rotate their plantings yearly

TO receive governmental price-support benefits for a crop farmers must have produced that same crop for the past several years

The statements above if true best support which of the following conclusions?

(A) The rules for governmental support of farm prices work against efforts to reduce water pollution

(B) The only solution to the problem of water pollution from fertilizers and pesticides is to take farmland out of production

(C) Farmers can continue to make a profit by rotating diverse crops thus reducing costs for chemicals but not by planting the same crop each yean

(D) New farming techniques will be developed to make it possible for farmers to reduce the application of fertilizers and pesticides

(E) Governmental price supports for farm products are set at levels that are not high enough to allow farmers to get out of debt

Hardin argued that grazing land held in common (that is open to any user)would always be used less carefully than private grazing land Each rancher would be tempted to overuse common land because the benefits would accrue to the individual while the costs of reduced land quality that result from overuse would be spread among all users But a study comparing 217 million acres of common grazing land with 433 million acres of private grazing land showed that the common land was in better condition

The answer to which of the following questions would be most useful in evaluating the significance in relation to Hardin's claim of the study described above?

(A) Did any of the ranchers whose land was studied use both common and private land?

(B) Did the ranchers whose land was studied tend to prefer using common land over using private land for grazing?

(C) Was the private land that was studied of comparable quality to the common land before either was used for grazing?

(D) Were the users of the common land that was studied at least as prosperous as the users of the private land?

(E) Were there any owners of herds who used only common land and no private land for grazing?

Which of the following if true and known by the ranchers would best help explain the results of the study?

(A) With private grazing land both the costs and the benefits of overuse fall to the individual user

(B) The cost in reduced land quality that is attributable to any individual user is less easily measured with common land than it is with private land

(C) An individual who overuses common grazing land might be able to achieve higher returns than other users can with the result that he or she would obtain a competitive advantage

(D) If one user of common land overuses it even slightly, the other users are likely to do so even more with the consequence that the costs to each user outweigh the benefits

(E) There are more acres of grazing land held privately than there are held in common

67. A compelling optical illusion called the illusion of velocity and size makes objects appear to be moving more slowly the larger the objects are Therefore a motorist s estimate of the time available for crossing a highway with a small car approaching is bound to be lower than It would be with a large truck approaching

The conclusion above would be more properly drawn if it were made clear that the 

(A) truck's speed is assumed to be lower than the car's

(B) truck's speed is assumed to be the same as the car's

(C) truck's speed is assumed to be higher than the car's

(D) motorist's estimate of time available is assumed to be more accurate with cars approaching than with trucks approaching

(E) motorist's estimate of time available is assumed to be more accurate with trucks approaching than with cars approaching

Advocates of a large-scale space-defense research project conclude that it will represent a net benefit to civilian business They say that since government-sponsored research will have civilian applications civilian businesses will reap the rewards of government-developed technology

Each of the following if true raises a consideration arguing against the conclusion above EXCEPT

(A) The development of cost-efficient manufacturing techniques is of the highest priority for civilian business and would be neglected if resources go to military projects which d0 not emphasize cost efficiency

(B) Scientific and engineering talent needed by civilian business will be absorbed by the large scale project

(C) Many civilian businesses will receive subcontracts to provide materials and products needed by the research project

(D) If government research money is devoted to the space project it will not be available for specifically targeted needs of civilian business where it could be more efficiently used.

(E) The increase in taxes or government debt needed to finance the project will severely reduce the vitality of the civilian economy

69. Manufacturers sometimes discount the price of a product to retailers for a promotion period when the product is advertised to consumers Such promotions often result in a dramatic increase in amount of product sold by the manufacturers to retailers Nevertheless the manufacturers could often make more profit by not holding the promotions

Which of the following if true most strongly supports the claim above about the manufacturers profit?

(A) The amount of discount generally offered by manufacturers to retailers is carefully calculated to represent the minimum needed to draw consumers attention to the product

(B) For many consumer products the period of advertising discounted prices to consumers is about a week not sufficiently long for consumers to become used to the sale price

(C) For products that are not newly introduced the purpose of such promotions is to keep the products in the minds of consumers and to attract consumers who are currently using competing products

(D) During such a promotion retailers tend to accumulate in their warehouses inventory bought at discount they then sell much of it later at their regular price.

(E) If a manufacturer fails to offer such promotions but its competitor offers them that competitor will tend to attract consumers away from the manufacturers product.

70. When people evade income taxes by not declaring taxable income a vicious cycle results Tax evasion forces lawmakers to raise income tax rates Which causes the tax burden on nonevading taxpayers to become heavier. This in turn encourages even more taxpayers to evade income taxes by hiding taxable income

The vicious cycle described above could not result unless which of the following were true?

(A) An Increase in tax rates tends to function as an incentive for taxpayers to try to increase their pretax incomes

(B) Some methods for detecting tax evaders, and thus recovering some tax revenue lost through evasion bring in more than they cost but their success rate varies from year to year.

(C) When lawmakers establish income tax rates in order to generate a certain level of revenue, they do not allow adequately for revenue that will be lost through evasion

(D) No one who routinely hides some taxable income can be induced by a lowering of tax rates to stop hiding such income unless fines for evaders are raised at the same time

(E) Taxpayers do not differ from each other with respect to the rate of taxation that will cause them to evade taxes

Because postage rates are rising Home Decorator magazine plans to maximize its profits by reducing by one-half the number of issues it publishes each year The quality of articles the number of articles published per year, and the subscription price will not change Market research shows that neither subscribers nor advertisers will be lost if the magazine's plan is instituted

Which of the following if true provides the strongest evidence that the magazine's profits are likely to decline if the plan is instituted?

(A) With the new postage rates a typical issue under the proposed plan would cost about one-third more to mail than a typical current issue would

(B) The majority of the magazine's subscribers are less concerned about a possible reduction in the quantity of the magazine's articles than about a possible IOSS of the current high quality of its articles

(C) Many of the magazine's long-time subscribers would continue their subscriptions even if the subscription price were increased

(D) Most of the advertisers that purchase advertising space In the magazine will continue to spend the same amount on advertising per issue as they have in the past

(E) Production costs for the magazine are expected to remain stable

72.A discount retailer of basic household necessities employs thousands of people and pays most of them at the minimum wage rate Yet following a federally mandated increase of the minimum wage rate that increased the retailer's operating costs considerably, the retailer's profits increased markedly

Which of the following if true most helps to resolve the apparent paradox?

(A) Over half of the retailer's operating costs consist of payroll expenditures yet only a small percentage of those expenditures go to pay management salaries

(B) The retailers customer base is made up primarily of people who earn or who depend on the earnings of others who earn the minimum wage

(C) The retailer's operating costs other than wages increased substantially after the increase in the minimum wage rate went into effect

(D) When the increase in the minimum wage rate went into effect the retailer also raised the wage rate for employees who had been earning just above minimum wage

(E) The majority of the retailer's employees work as cashiers and most cashiers are paid the minimum wage

73. The cotton farms of Country became so productive that the market could not absorb all that they produced Consequently, cotton prices fell The government tried to boost cotton prices by offering farmers who took 25 percent of their cotton acreage out of production direct support payments up to a specified maximum per farm

The government's program if successful will not be a net burden on the budget Which of the following if true is the best basis for an explanation of how this could be so?

(A) Depressed cotton prices meant operating losses for cotton farms and the government lost revenue from taxes on farm profits

(B) Cotton production in several countries other than Q declined slightly the year that the support-payment program went into effect in Q

(C) The first year that the support-payment program was in effect cotton acreage in Q was 5 percent below its level in the base year for the program

(D) The specified maximum per farm meant that for very large cotton farms the support payments were less per acre for those acres that were withdrawn from production than they were for smaller farms

(E) Farmers who wished to qualify for support payments could not use the cotton acreage that was withdrawn from production to grow any other crop

7 United States hospitals have traditionally relied primarily on revenues from paying patients to offset losses from unreimbursed care Almost all paying patients now rely on governmental or private health insurance to pay hospital bill. Recently, insurers have been strictly limiting what they pay hospitals for the care of insured patients to amounts at or below actual costs

Which of the following conclusions is best supported by the information above?

(A) Although the advance of technology has made expensive medical procedures available to the wealthy, such procedures are out of the reach of low-income patients

(B) If hospitals do not find ways of raising additional Income for unreimbursed care they must either deny some of that care or suffer losses if they give it

(C) Some patients have incomes too high for eligibility for governmental health insurance but are unable to afford private insurance for hospital care

(D) If the hospitals reduce their costs in providing care insurance companies will maintain the current level of reimbursement thereby providing more funds for unreimbursed care

(E) Even though philanthropic donations have traditionally provided some support for the hospitals such donations are at present declining

Generally scientists enter their field with the goal of doing important new research and accept as their colleagues those with similar motivation Therefore when any scientist wins renown as an expounder of science to general audiences most other scientists conclude that this popularizer should no longer be regarded as a true colleague

The explanation offered above for the low esteem in which scientific popularizers are held by research scientists assumes that

(A) serious scientific research is not a solitary activity, but relies on active cooperation among a group of colleagues

(B) research scientists tend not to regard as colleagues those scientists whose renown they envy

(C) a scientist can become a famous popularizer without having completed any important research

(D) research scientists believe that those who are well known as popularizers of science are not motivated to do important new research

(E) no important new research can be accessible to or accurately assessed by those who are not themselves scientists

76. Country Y uses its scarce foreign-exchange reserves to buy scrap iron for recycling into steel Although the steel thus produced earns more foreign exchange than it costs that policy is foolish Country Y's own territory has vast deposits of iron ore which can be mined with minimal expenditure of foreign exchange

Which of the following if true provides the strongest support for Country Y's policy of buying scrap iron abroad?

(A) The price of scrap iron on international markets rose significantly in 1987

(B) Country Y s foreign exchange reserves dropped significantly in 1987

(C) There is virtually no difference in quality between steel produced from scrap iron and that produced from iron ore

(D) Scrap iron is now used in the production of roughly half the steel used in the world today, and experts predict that scrap iron will be used even more extensively in the future

(F) Furnaces that process scrap iron can be built and operated in Country Y with substantially less foreign exchange than can furnaces that process iron ore

77. Parasitic wasps lay their eggs directly into the eggs of various host insects in exactly the right numbers for any suitable size of host egg If they laid too many eggs in a host egg the developing wasp larvae would compete with each other to the death for nutrients and space If too few eggs were laid portions of the host egg would decay, killing the wasp larvae

Which of the following conclusions can properly be drawn from the information above?

(A) The size of the smallest host egg that a wasp could theoretically parasitize can be determined from the wasp's egg-laying behavior.

(B) Host insects lack any effective defenses against the form of predation practiced by parasitic wasps

(C) Parasitic wasps learn from experience how many eggs to lay into the eggs of different host species

(D) Failure to lay enough eggs would lead to the death of the developing wasp larvae more quickly than would laying too many eggs

(E) Parasitic wasps use visual clues to calculate the size of a host egg

Consumer health advocate Your candy company adds caffeine to your chocolate candy bars so that each one delivers a specified amount of caffeine. Since caffeine is highly addictive this indicates that you intend to keep your customers addicted

Candy manufacturer Our manufacturing process results in there being less caffeine in each chocolate candy bar than in the unprocessed cacao beans from which the chocolate is made

The candy manufacturer's response is flawed as a refutation of the consumer health advocate's argument because it

(A) fails to address the issue of whether the level of caffeine in the candy bars sold by the manufacturer is enough to keep people addicted

(B) assumes without warrant that all unprocessed cacao beans contain a uniform amount of caffeine

(C) does not specify exactly how caffeine is lost in the manufacturing process

(D) treats the consumer health advocates argument as though it were about each candy bar rather than about the manufacturers candy in general

(E) merely contradicts the consumer health advocate's conclusion without giving any reason to believe that the advocates reasoning is unsound

To evaluate a plan to save money on office-space expenditures by having its employees work at home XYZ Company asked volunteers from its staff to try the arrangement for six months During this period the productivity of these employees was as high as or higher than before

Which of the following if true would argue most strongly against deciding on the basis of the trial results to implement the company's plan?

(A) The employees who agreed to participate In the test of the plan were among the company's most self-motivated and independent workers

(B) The savings that would accrue from reduced office-space expenditures alone would be sufficient to justify the arrangement for the company, apart from any productivity increases

(C) Other companies that have achieved successful results from work-at-home plans have work forces that are substantially larger than that of XYZ

(D) The volunteers who worked at home were able to communicate with other employees as necessary for performing the work.

(E) Minor changes in the way office work is organized at XYZ would yield increases in employee productivity similar to those achieved in the trial

80. Vitacorp, a manufacturer, wishes to make its information booth at an industry convention more productive in terms of boosting sales The booth 0ffers information introducing the company's new products and services To achieve the desired result Vitacorp's marketing department will attempt to attract more people to the booth The marketing director's first measure was to instruct each salesperson to call his or her five best customers and personally invite them to visit the booth

Which of the following if true most strongly supports the prediction that the marketing director's first measure will contribute to meeting the goal of boosting sales?

(A) Vitacorp's salespeople routinely inform each important customer about new products and services as soon as the decision to launch them has been made

(B) Many of Vitacorp's competitors have made plans for making their own information booths more productive in increasing sales

(C)An information booth that is well attended tends to attract visitors who would not otherwise have attended the booth

(D) Most of Vitacorp's best customers also have business dealings with Vitacorp's competitors

(E) Vitacorp has fewer new products and services available this year than it had in previous years

81. An eyeglass manufacturer tried to boost sales for the summer quarter by offering its distributors a special discount if their orders for that quarter exceeded those for last year's summer quarter by at least 20 percent Many distributors qualified for this discount Even with much merchandise discounted sales increased enough to produce a healthy gain in net profits The manufacturer plans to repeat this success by offering the same sort of discount for the fall quarter

Which of the following if true most clearly points to a flaw in the manufacturer's plan to repeat the successful performance of the summer quarter?

(A) In general, a distributor's orders for the summer quarter are no higher than those for the spring quarter

(B) Along with offering special discounts to qualifying distributors the manufacturer increased newspaper and radio advertising in those distributors sales areas

(C) The distributors most likely to qualify for the manufacturer's special discount are those whose orders were unusually low a year earlier.

(D) The distributors who qualified for the manufacturer's special discount were free to decide how much of that discount to pass on to their own customers

(E) The distributors ordering more goods in the summer quarter left them overstocked for the fall quarter

Consumer advocate It is generally true at least in this state that lawyers who advertise a specific service charge less for that service than lawyers who do not advertise It is also true that each time restrictions on the advertising of legal services have been eliminated the number of lawyers advertising their services has increased and legal costs to consumers have declined in consequence However, eliminating the state requirement that legal advertisements must specify fees for specific services would almost certainly increase rather than further reduce consumers legal costs Lawyers would no longer have an incentive to lower their fees when they begin advertising and if no longer required to specify fee arrangements many lawyers who now advertise would increase their fees

In the consumer advocate's argument the two portions in boldface play which of the following roles?

(A) The first is a generalization that the consumer advocate accepts as true the second is presented as a consequence that follows from the truth of that generalization

(B) The first is a pattern of cause and effect that the consumer advocate argues will be repeated in the case at issue the second acknowledges a circumstance in which that pattern would not hold

(C) The first is a pattern of cause and effect that the consumer advocate predicts will not hold in the case at issue the second offers a consideration in support of that prediction

(D) The first is evidence that the consumer advocate offers in support of a certain prediction the second is that prediction

(E) The first acknowledges a consideration that weighs against the main position that the consumer advocate defends the second is that position

5 Critical Reasoning Answer Key

1A

32E

63E

2D

33C

64A

3B

34C

65C

4A

35E

66D

5C

36A

67B

6C

37D

68C

7D

38B

69D

8D

39B

70C

9A

40C

71D

10B

41E

72B

11B

42B

73A

12C

43C

74B

13E

44E

75D

14B

45E

76E

15A

46B

77A

16D

47D

78A

17A

48E

79A

18A

49C

80C

19A

50D

81E

20C

51C

82C

21E

52A

22D

53B

23B

54C

24A

55A

25C

56E

26C

57E

27B

58D

28E

59C

29B

60B

30B

61A

31D

62C

6 Critical Reasoning Answer Explanations

The following discussion is intended to familiarize you with the most efficient and effective approaches to critical reasoning questions The particular questions in this chapter are generally representative of the kinds of critical reasoning questions you will encounter on the GMAT Remember that it is the problem solving strategy that is important, not the specific details of a particular question.

Which of the following best completes the passage below?

In a survey of job applicants two fifths admitted to being at least a little dishonest However, the survey may underestimate the proportion of job applicants who are dishonest because 

(A) some dishonest people taking the survey might have claimed on the survey to be honest

(B) some generally honest people taking the survey might have claimed on the survey to be dishonest

(C) some people who claimed on the survey to be at least a little dishonest may be very dishonest

(D) some people who claimed on the survey to be dishonest may have been answering honestly

(E) some people who are not job applicants are probably at least a little dishonest

Argument Evaluation

Situation Survey asking about dishonesty may underestimate dishonesty

Reasoning How could the survey present an underestimate? Accurate survey results depend on honest answers If some dishonest people say they are honest then the real number of dishonest people is actually higher than the survey states The survey therefore may underestimate how many people are dishonest

A Correct This statement properly identifies the error in the survey method

B This leads to an overestimate not an underestimate

C The degree of dishonesty is irrelevant

D This 1eads to a correct estimate not an underestimate

E People who are not job applicants are irrelevant

The correct answer is A

2. Increases in the level of high-density lipoprotein(HDL)in the human bloodstream lower bloodstream cholesterol levels by increasing the body-capacity to rid itself of excess cholester01 Levels of HDL in the bloodstream of some individuals are significantly increased by a program of regular exercise and weight reduction

Which of the following can be correctly inferred from the statements above?

(A) Individuals who are underweight do not run any risk of developing high levels of cholesterol in the bloodstream

(B) Individuals who do not exercise regularly have a high risk of developing high levels of cholesterol in the bloodstream late in life

(C) Exercise and weight reduction are the most effective methods of lowering bloodstream cholesterol levels in humans

(D) A program of regular exercise and weight reduction lowers cholesterol levels in the bloodstream of some individuals

(E) Only regular exercise is necessary to decrease cholesterol levels in the bloodstream of individuals of average weight

Argument Construction

Situation Higher HDL levels in the bloodstream reduce cholestero1 Regular exercise and weight reduction promote higher HDL levels in some people

Reasoning What inference is supported by this information ? The first statement is a general one applying to all people The second one applies only to some people The resulting inference can be made only about some people not everyone Because some people achieve higher HDL levels through a program of regular exercise and weight reduction these individuals will have lower cholesterol levels

A The passage draws no comparison between being underweight and having lower cholesterol levels

B The passage does not discuss lack of regular exercise as a risk factor for the development of high bloodstream cholesterol late in life

C Other possible methods of lowering cholesterol 1evels arc not discussed and so a program of exercise and weight reduction cannot be inferred to be the best method Moreover, a general inference applying to all humans cannot be made of the basis of some individuals

D Correct This statement properly identifies the inference that since a program of exercise and weight reduction raises HDL for some people that program should lower cholesterol for some people

E The passage explicitly states that the two elements of regular exercise and weight reduction together contribute to some individuals ability to increase their HDL 1evels It cannot be inferred that all individuals of average weight can lower their cholesterol with regular exercise alone

The correct answer is D

3. A cost-effective solution to the problem of airport congestion is to provide high-speed ground transportation between major cities lying 200 to 500 miles apart The successful implementation of this plan would cost far less than expanding existing airports and would also reduce the number of airplanes clogging both airports and

Which of the following if true could proponents of the plan above most appropriately cite as a piece of evidence for the soundness of their plan?

(A) An effective high speed ground-transportation system would require major repairs to many highways and mass-transit improvements

(B) One-half of all departing flights in the nation's busiest airport head for a destination in a major city 225 miles away

(C) The majority of travelers departing from rural airports are flying to destinations in cities over 600 miles away

(D) Many new airports are being built in areas that are presently served by highs peed ground-transportation systems

(E) A large proportion of air travelers are vacationers who are taking long-distance flights.

Evaluation of a Plan

Situation Providing high-speed ground transportation between cities 200 to 500 miles apart is a more cost-effective and efficient way to reduce airport congestion than expanding existing airports.

Reasoning Wheat evidence support the plan? The transportation plan will work only if people are likely to use the high-speed ground transportation If halt the flights leaving the busiest airport fly to destinations within range(200-500 miles)of ground transportation then many people might choose it over air travel Fewer flights would be needed and airport congestion would decrease.

A Expensive repairs are an argument against not for, ground transportation.

B Correct This statement properly identifies evidence that supports the plan.

C Rural travelers are not included in this proposal which addresses travel between major cities.

D New airports covering the same routes are a threat to the plan.

E The information that the airports are congested because of long-distance travelers rather than those heading to destinations within 500 miles argues against the proposal.

The correct answer is B

The price the government pays for standard weapons purchased from military contractors is determined by a pricing method called historical costing Historical costing allows contractors to protect their profits by adding a percentage increase based on the current rate of inflation to the previous years contractual price

Which of the following statements if true is the best basis for a criticism of historical costing as an economically sound pricing method for military contracts?

(A) The government might continue to pay for past inefficient use of funds

(B) The rate of inflation has varied considerably over the past twenty years

(C)The contractual price will be greatly affected by the cost of materials used for the products

(D) Many taxpayers question the amount of money the government spends on military contracts

(E) The pricing method based on historical costing might not encourage the development of innovative weapons

Argument Evaluation

Situation Military contractors add a percentage increase based on the rate of inflation to prices specified in previous contracts for standard weapons

Reasoning What could wrong in simply adding a percentage increase each year? If the original contract price accommodated the contractors inefficiencies the government's overpayments for these inefficiencies are simply perpetuated and money continues to be wasted Additionally, historical costing assumes costs only go up never down

A Correct This statement properly identifies a serious problem with historical costing

B Pointing out the variable rate of inflation shows a strength rather than a weakness of historical costing

C Identifying the importance of the cost of materials which also varies with inflation is a strength of the method

D Taxpayers concerns are outside the scope of this question

E The contract is for standard weapons only innovative weapons are not discussed

The correct answer is A

5. Which of the following best completes the passage below? Established companies concentrate on defending what they already have Consequently, they tend not to be innovative themselves and tend to underestimate the effects of the innovations of others The clearest example of this defensive strategy is the fact that 

(A) ballpoint pens and soft-tip markers have eliminated the traditional market for fountain pens clearing the way for the marketing of fountain pens as luxury or prestige items

(B) a highly successful automobile was introduced that had been a dismal failure

by the same company that had earlier introduced a model

(C) a once-successful manufacturer of slide rules reacted to the introduction of electronic calculators by trying to make better slide rules

(D) one of the first models of modern accounting machines designed for use in the banking industry, was purchased by a public library as well as by banks

(E) the inventor of a commonly used anesthetic did not intend the product to be used by dentists who currently account for almost the entire market for that drug

Argument Construction

Situation Established companies focus on defending their existing products rather than seeking opportunities for innovation They also underestimate the importance of other companies innovations

Reasoning Which example clearly shows the use of a defensive strategy? A company is using a defensive strategy when it limits its business focus strictly to the improvement of an existing product instead of recognizing and responding to a significant innovation The producer of slide rules who reacts to the introduction of electronic calculators by trying to make better slide rules is a perfect example of the failure of a company to respond appropriately to innovation

A Fountain pens have not been rendered obsolete by innovations in writing instruments fountain-pen producers responded to the innovations by redefining the market for their product

B The automobile company did not respond defensively and simply reproduce the same failed model it produced a new, successful model

C Correct This statement properly identifies an example of defensive strategy

D A product's ability to appeal to an unintended audience is not an example of a defensive strategy

E The anesthetic finds a different market than its inventor anticipated this is not a defensive strategy

The correct answer is C

Since the mayors publicity campaign for Greenville's bus service began six months ago morning automobile traffic into the midtown area of the city has decreased seven percent. During the same period there has been an equivalent rise in the number of persons riding buses into the midtown area Obviously, the mayor's publicity campaign has convinced many people to leave their cars at home and ride the bus to work

Which of the following if true casts the most serious doubt on the conclusion drawn above?

(A) Fares for all bus mutes in Greenville have risen an average of five percent during the past six months

(B) The mayor of Greenville rides the bus to City Hall in the city's midtown area

(C) Road reconstruction has greatly reduced the number of lanes available to commuters in major streets leading to the midtown area during the past six months

(D) The number of buses entering the midtown area of Greenville during the morning hours is exactly the same now as it was one year ago

(E) Surveys show that longtime bus riders are no more satisfied with the Greenville bus service than they were before the mayors publicity campaign began

Argument Evaluation

Situation Traffic into midtown has decreased by seven percent and bus rider ship has increased by an equivalent amount The mayor's publicity campaign is responsible for this change

Reasoning What casts doubt on this conclusion? Another reasonable cause for the decrease in automobile traffic and the increase in bus rider ship would make this conclusion suspect Road construction impeding access to midtown over the same period of time is a reasonable alternative explanation The road construction projects would likely have caused fewer people to drive to midtown many of these people have probably taken the bus

A An increase in fares might be a reasonable explanation for decreased rather than this increased rider ship

B The mayors decision to ride the bus sets a good example for citizens and his example tends to strengthen the conclusion

C Correct This statement properly identifies an explanation that weakens the conclusion

D An alternative explanation based on the running of more buses can be eliminated

E Passengers perceive bus service to be the same so better service can be eliminated as a possible cause of the increased rider ship

The correct answer is C

A researcher discovered that people who have low levels of immune system activity tend to score much lower on tests of mental health than do people with normal or high immune system activity The researcher concluded from this experiment that the immune system protects against mental illness as well as against physical disease

The researcher's conclusion depends on which of the following assumptions?

(A) High immune system activity protects against mental illness better than normal immune, system activity does

(B) Mental illness is similar to physical disease in its effects on body systems.

(C) People with high immune-system activity cannot develop mental illness

(D) Mental illness does not cause people's immune-system activity to decrease.

(E) Psychological treatment of mental illness is not as effective as is medical treatment.

Argument Construction

Situation Finding that people with low immune-system activity score lower on mental health tests than people with normal or high levels of immune activity, a researcher concludes that the immune system protects mental as well as physical health.

Reasoning What assumption does the researcher make? The researcher asserts that immune-system activity can inhibit mental illness as it does physical illness While it is possible that mental i11ness might itself depress immune-system activity, the researcher assumes that this is not the case If mental illness caused a decline in immune-system activity, then lower levels of immune-system activity would be expected as a result and the higher levels would merely indicate the absence of illness rather than any effect on illness.

A The researcher does not distinguish between high and normal levels of immune-system activity, so this assumption is not needed

B The researcher is investigating only mental illness not physical disease so this assumption is not needed

C Immune-system activity could protect mental health without offering total prevention of mental illness this assumption is not needed

D Correct This statement properly identifies does not decrease immune system activity the researcher's underlying assumption that mental illness

E Since different treatments are not discussed any assumption about them is unnecessary

The correct answer is D

8. It is true that it is against international law to sell plutonium to countries that do not yet have nuclear weapons But if United States companies do not do so companies in other countries will

Which of the following is most like the argument above in its logical structure?

(A) It is true that it is against the police department's policy to negotiate with kidnappers But if the police want to prevent loss of life they must negotiate in some cases

(B) It is true that it is illegal to refuse to register for military service But there is a long tradition in the United States of conscientious objection to serving in the armed forces

(C) It is true that it is illegal for a government official to participate in a transaction in which there is an apparent conflict of interest But If the facts are examined carefully, it will clearly be seen that there was no actual conflict of interest in the defendant's case

(D) It is true that it is against the law to burglarize people's homes But someone else certainly would have burglarized that house If the defendant had not done so first

(E) It is true that company policy forbids supervisors to fire employees without two written warnings But there have been many supervisors who have disobeyed this policy

Argument Construction

Situation If American companies comply with international law and refuse to sell plutonium to countries without nuclear weapons companies in other countries will go ahead and sell plutonium illegally

Reasoning Which argument has the same logical structure as the argument about selling plutonium? Consider how the argument proceeds The plutonium argument first identifies a law Then it makes an excuse for breaking that law by asserting that the law will inevitably be broken by others in the future This implies that it is acceptable to break the law now because it will be broken sometime

A This argument states a reason for breaking the law, preventing loss of life but it does not say that the law will inevitably be broken in the future

B This argument acknowledges the law has been broken in the past it says nothing about the future

C This argument shows that the law does not apply to a particular case

D Correct This statement properly identifies an argument that is parallel in logical structure to the plutonium argument because it too asserts the inevitability that the 1aw will be broken at some time by someone and excuses the defendant as merely being the first to do so

E This argument demonstrates that a policy has been previously disobeyed it does not state that it will be disobeyed in the future

The correct answer is D

9. Extinction is a process that can depend on a variety of ecological geographical and physiological variables. These variables affect different species of organisms in different ways and should therefore yield a random pattern of extinctions However, the fossil record shows that extinction occurs in a surprisingly definite pattern with many species vanishing at the same time

Which of the following if true forms the best basis for at least a partial explanation of the patterned extinctions revealed by the fossil record?

(A) Major episodes of extinction can result from widespread environmental disturbances that affect numerous different species

(B) Certain extinction episodes selectively affect organisms with particular sets of characteristics unique to their species

(C) Some species become extinct because of accumulated gradual changes in their local environments.

(D) In geologically recent times for which there is no fossil record human intervention has changed the pattern of extinctions

(E) Species that are widely dispersed are the least likely to become extinct.

Argument Construction

Situation The fossil record reveals that species become extinct in a surprisingly definite pattern with multiple species vanishing simultaneously

Reasoning Which point provides a basis for explaining the pattern? The passage states that the process of extinction depends on so many variablesin the ecology and geography of the environment and in the physiology of the species-that the expected outcome would be a random pattern of extinctions Yet a definite pattern is found instead What could explain the disappearance of multiple species at the same time? What if the magnitude of one variable is so overwhelming that it prevails over the impact of the other variables? It is possible that significant widespread changes in the environment might affect multiple species simultaneously, causing their extinction

A Correct This statement properly identifies a basis for explaining the pattern of many species becoming extinct simultaneously

B This explanation of selective extinction does not explain how many species become extinct at the same time

C This explanation addresses only some species not many species

D The passage is based on what the fossil record suggests more recent times having no fossil record are outside the consideration of the passage

E Indicating which species are least likely to become extinct does not explain a pattern of simultaneous extinction of many species

The correct answer is A

Which of the following best completes the passage below?

At a recent conference on environmental threats to the Noah Sea most participating countries favored uniform controls on the quality of effluents whether or not specific environmental damage could be attributed to a particular source of effluent What must, of course, be shown, in order to avoid excessively restrictive controls is that 

(A) any uniform controls that are adopted are likely to be implemented without delay

(B) any substance to be made subject to controls can actually cause environmental damage

(C) the countries favoring uniform controls are those generating the largest quantities of effluents

(D) all of any given pollutant that is to be controlled actually reaches the North Sea at present

(E) environmental damage already inflicted on the North Sea is reversible

Argument Construction

Situation In the face of environmental threats to the North Searestrictions on effluents are considered

Reasoning How can excessively restrictive controls be avoided? To prevent pollutants from entering the North Sea countries decide to control the quality of effluents They need to control only those effluents that cause environmental damage There is no need to restrict harmless effluents

A The immediacy of adopting controls does not prevent the controls from being overly restrictive

B Correct This statement properly identifies the fact that controls on harmless effluents would be excessively restrictive and so should be avoided

C Avoiding unnecessary restrictions involves analyzing the quality of the effluents not the composition of the countries favoring the restrictions

D It is not necessary to prove that all of a pollutant reaches the North Sea It is necessary to prove only that some of it does

E The environmental damage that has already been caused is outside the scope of the restrictions Finding that the damage is reversible will do nothing to prevent unnecessary restrictions

The correct answer is B

Shelby Industries manufactures and sells the same gauges as Jones Industries Employee wages account for forty percent of the cost of manufacturing gauges at both Shelby Industries and Jones Industries. Shelby Industries is seeking a competitive advantage over Jones Industries Therefore to promote this end Shelby Industries should lower employee wages

Which of the following if true would most weaken the argument above?

(A) Because they make a small number of precision instruments gauge manufacturers cannot receive volume discounts on raw materials

(B) Lowering wages would reduce the quality of employee work and this reduced quality would lead to lowered sales

(C) Jones Industries has taken away twenty percent of Shelby Industries business over the last yean

(D) Shelby Industries pays its employees on average ten percent more than does Jones Industries.

(E) Many people who work for manufacturing plants five in areas in which the manufacturing plant they work for is the only industry

Evaluation of a Plan

Situation Two industries manufacture the same product and employee wages at both industries account for the same percentage of the manufacturing cost One company seeks a competitive edge by lowering employee wages

Reasoning What point weakens the argument in favor of lowering wages? The company anticipates that the result of lowering wages will be a competitive edge presumably gained through offering its products at 10wer prices as the result of having 1ower production costs If evidence can be shown that lowering wages will not result in gaining a competitive advantage the argument is weakened If instead the result of lowered wages is lowered employee morale and thus a 1ower quality of work and products the declining quality could 1ead to a loss of sales In this case the action would not only fan to provide a competitive edge it would put the company and its products at a disadvantage

A Because the company is unable to get lower costs on its raw materials it is more likely seek other ways of lowering its costs such as reducing wages This statement tends to support the argument.

B Correct This statement properly identifies a factor that weakens the argument.

C The loss of business would only encourage the Shelby company to seek a competitive edge perhaps by cutting employee wages so this statement does not weaken the conclusion

D Its relatively higher employee wages show only why the company might reduce wages the conclusion about gaining a competitive advantage is not weakened

E Because of the location of the company, employees are likely to continue working even if wages are lowered this statement strengthens rather than weakens the conclusion

The correct answer is B

12. Large national budget deficits do not cause large trade deficits If they did countries with the largest budget deficits would also have the largest trade deficits In fact when deficit figures are adjusted so that different countries are reliably comparable to each other, there is no such correlation

If the statements above are all true which of the following can properly be inferred on the basis of them?

(A) Countries with large national budget deficits tend to restrict foreign trade

(B) Reliable comparisons of the deficit figures of one country with those of another are impossible

(C) Reducing a country's national budget deficit will not necessarily result in a lowering of any trade deficit that country may have

(D) When countries are ordered from largest to smallest in terms of population the smallest countries generally have the smallest budget and trade deficits

(E) Countries with the largest trade deficits never have similarly large national budget deficits

Argument Construction

Situation No correlation is found between large national budget deficits and large trade deficits

Reasoning What inference can be drawn from this information? The inference must be based on the stated information Since the passage states that national budget deficits are not tied to trade deficits it is logical to anticipate an inference about the independent nature of the relationship between the two kinds of deficits One possible inference is that reducing one deficit has no impact on the reduction of the other

A An inference about restricting foreign trade cannot be drawn because no information about such restrictions is given

B The passage states that reliable comparisons have been developed

C Correct This statement properly identifies an inference that can be drawn from the given information

D There is no information in the passage about population size so no inference about it can be drawn

E The two kinds of deficits are independent of each other it cannot be inferred that such independence prohibits both deficits from being large

The correct answer is C

13. A famous singer recently won a lawsuit against an advertising firm for using another singer in a commercial to evoke the famous singer's well-known rendition of a certain song As a result of the lawsuit advertising firms will stop using imitators in commercials Therefore advertising costs will rise since famous singers services cost more than those of their imitators

The conclusion above is based on which of the following assumptions?

(A) Most people are unable to distinguish a famous singer's rendition of a song from a good imitator's rendition of the same song

(B) Commercials using famous singers are usually more effective than commercials using imitators of famous singers

(C) The original versions of some well-known songs are unavailable for use in commercials

(D) Advertising firms will continue to use imitators to mimic the physical mannerisms of famous singers

(E) The advertising industry will use well-known renditions of songs in commercials Argument Construction

Situation A famous singer successfully sues an advertising company for using an imitator of that singer in a commercial Advertising costs will now rise because the fees of famous singers are higher than the fees of their imitators

Reasoning What assumption must be true to support the argument that costs will rise? Consider what specifically is assumed to be driving the rise in advertising costs It is not the fact of an increased production cost for a certain dement of the commercials it is the choice of those who make decisions about what elements to include in their commercials The assumption is that despite the outcome of the suit the companies will still want to use well-known versions of songs in their commercials and thus will have to hire the famous singers of those songs and pay their higher fees

A Whether or not the audience can tell the difference between the singers is beside e point the audience s listening aptitude does not cause advertising costs to rise

B The effectiveness of the commercials is not in question the cause of the rising costs is the issue

C The lack of availability of some songs is not relevant to the rise in advertising costs The same songs would not have been available even when production costs were lower

D The passage explicitly states that after the lawsuit advertising firms will stop using imitators in commercials This statement contradicts the passage and cannot possibly be assumed

E  Correct This statement properly identifies assumption that underlies the conclusion that costs will rise

The correct answer is E

The sustained massive use of pesticides in farming has two effects that are especially pernicious First it often kills off the pests natural enemies in the area Second it often unintentionally gives rise to insecticide resistant pests since those insects that survive a particular insecticide will be the ones most resistant to it, and they are the ones left to breed

From the passage above it can be properly inferred that the effectiveness of the sustained massive use of pesticides can be extended by doing which of the following assuming that each is a realistic possibility?

(A) Using only chemically stable insecticides

(B) Periodically switching the type of insecticide used

(C) Gradually increasing the quantities of pesticides used

(D) Leaving a few fields fallow every year

(E) Breeding higher-yielding varieties of crop plants

Evaluation of a Plan

Situation Continued high level pesticide use often kills off the targeted pests natural enemies In addition the pests that survive the application of the pesticide may become resistant to it and these pesticide resistant pests will continue breeding

Reasoning What can be done to prolong the effectiveness of pesticide use? It can be inferred that the ongoing use of a particular pesticide will not continue to be effective against the future generations of pests with an inherent resistance to that pesticide What would be effective against these future generations? If farmers periodically change the particular pesticide they use then pests resistant to one kind of pesticide might be killed by another This would continue with pests being killed off in cycles as the pesticides are changed It is also possible that this rotation might allow some of the pests natural enemies to survive at least until the next cycle

A Not enough information about chemically stable insecticides is given to make a sound inference

B Correct This statement properly identifies an action that could extend the effectiveness of pesticide use

C Gradually increasing the amount of the pesticides being used will not help the situation since the pests are already resistant to it

D Continued use of pesticides is assumed as part of the argument Since pesticides would be unnecessary for fallow fields this suggestion is irrelevant

E Breeding higheryielding varieties of crops does nothing to extend the effectiveness of the use of pesticides

The correct answer is B

Treatment for hypertension forestalls certain medical expenses by preventing strokes and heart disease. Yet any money so saved amounts to only one-fourth of the expenditures required to treat the hypertensive population Therefore there is no economic justification for preventive treatment for hypertension.

Which of the following if true is most damaging to the conclusion above?

(A) The many fatal strokes and heart attacks resulting from untreated hypertension cause insignificant medical expenditures but large economic losses of other sorts

(B) The cost per patient of preventive treatment for hypertension would remain constant even if such treatment were instituted on a large scale

(C) In matters of health care economic considerations should ideally not be dominant

(D) Effective prevention presupposes early diagnosis and programs to ensure early diagnosis are costly

(E) The net savings in med l resources achieved by some preventive health measures are smaller than the net losses attributable to certain other measures of this kind

Argument Evaluation

Situation The preventive treatment of hypertension cannot be economically justified because the costs of treating the entire hypertensive population are far greater than the costs of treating the strokes and heart disease that some hypertensive patients might suffer

Reasoning Which evidence most preventive the argument ? The argument compares only two costs: the preventive treatment of hypertension and the treatment of strokes and heart disease. The conclusion is based solely on that comparison. What if there are other significant costs? If other large economic losses occur when hypertension goes untreated, then treatment may be economically justified, and the argument is damaged

A Correct This statement properly identifies evidence that damages the conclusion

B The passage asserts that preventive treatment costs would be too high that the cost would be constant neither changes that opinion nor damages the conclusion

C The conclusion can only be damaged by an objection that is framed in the same terms in this case in economic terms The opinion stated here is not relevant to the conclusion

D Since early diagnosis programs increase the costs of preventive care this statement supports rather than damages the conclusion

E This statement provides no specific information about hypertension other preventive health measures are irrelevant to the conclusion

The correct answer is A

In an attempt to promote the widespread use of paper rather than plastic and thus reduce non-biodegradable waste the council of a small town plans to ban the sale of disposable plastic goods for which substitutes made of paper exist The council argues that since most paper is entirely biodegradable paper goods are environmentally preferable

Which of the following if true indicates that the plan to ban the sale of disposable plastic goods is ill suited to the town council's environmental goals?

(A) Although biodegradable plastic goods are now available members of the town council believe biodegradable paper goods to be safer for the environment

(B) The paper factory at which most of the townspeople are employed plans to increase production of biodegradable paper goods

(C) After other towns enacted similar bans on the sale of plastic goods the environmental benefits were not discernible for several years

(D) Since most townspeople prefer plastic goods to paper goods in many instances they are likely to purchase them in neighboring towns where plastic goods are available for sale

(E) Products other than those derived from wood pulp are often used in the manufacture of paper goods that are entirely biodegradable

Evaluation of a Plan

Situation A town council considers banning the sale of disposable plastic goods for which there are paper substitutes because paper is biodegradable and therefore environmentally preferable

Reasoning What problem might there be in the council's plan? The plan is intended to reduce the amount of plastic used by the citizens of the town The town council's plan is however, only as effective as the support it has from the town's citizens If the citizens prefer disposable plastic goods and if the goods are also readily available in neighboring towns there is nothing to stop them from buying the plastic goods elsewhere if that is their preference

A The existence of biodegradable plastic does not make the town's plan to ban disposable plastic goods unsuitable to its environmental goal

B The town's plan is no less appropriate for its goal if the local paper factory makes biodegradable paper goods

C Environmental benefits need not be immediate Even though benefits were not perceived fir several years after similar bans the benefits did occur

D Correct This statement properly identifies the problem with the council's plan

E The specific materials that go into making no biodegradable paper are not in question This statement is irrelevant to the plan

The correct answer is D

Since the deregulation of airlines delays at the nation's increasingly busy airports have increased by 25 percent To combat this problem more of the takeoff and landing slots at the busiest airports must be allocated to commercial airlines

Which of the following if true casts the most doubt on the effectiveness of the solution proposed above?

(A) The major causes of delays at the nation's busiest airports are bad weather and overtaxed air traffic control equipment

(B) Since airline deregulation began the number of airplanes in operation has increased by 25 percent.

(C) Over 60 percent of the takeoff and landing slots at the nation's busiest airports are reserved for commercial airlines

(D) After a small Midwestern airport doubled its allocation of takeoff and landing slots the number of delays that were reported decreased by 50 percent

(E) Since deregulation the average length of delay at the nations busiest airports has doubled

Evaluation of a Plan

Situation To reduce delays more takeoff and landing slots at the busiest airports should go to commercial airlines

Reasoning What point casts the most doubt on the proposed solution? Evaluating the effectiveness of this solution means examining the relation between the problem an increase in delays and the solution an increase in takeoff and landing slots Could the delays be caused by other factors that the solution fails to address? If the major causes for delays are related to weather and air traffic control equipment not to the number of slots for takeoff and landing, then the proposed solution of changing the allocations of the slots is bound to be less effective

A Correct This statement properly identifies a weakness in the proposed solution

B The increasing number of planes shows why airports are busier Since there is no information about whether these planes are private or commercial, this statement neither casts doubt on the solution nor supports it

C This statement suggests that there are slots available to be allocated to commercial airlines and thus offers some support for the proposed solution

D The example of one airport shows that the solution may work

E The increase in length of delay shows the scope of the problem this fact does not cast doubt on the solution

The correct answer is A

18.Unlike the wholesale price of raw wool the wholesale price of raw cotton has fallen considerably in the last yean Thus although the retail price of cotton clothing at retail clothing stores has not yet fallen it will inevitably fall

Which of the following if true most seriously weakens the argument above?

(A) The cost of processing raw cotton for cloth has increased during the last yean

(B) The wholesale price of raw wool is typically higher than that of the same volume of raw cotton

(C) The operating costs of the average retail clothing store have remained constant during the last year

(D) Changes in retail prices always lag behind changes in wholesale prices

(E) The cost of harvesting raw cotton has increased in the last yean

Argument Evaluation

Situation Since the wholesale price of raw cotton has fallen significantly, the retail price of cotton clothing in stores will inevitably fall

Reasoning What point weakens this argument? Consider carefully the difference between the two products for which costs are being compared cotton and cloth This argument assumes that lower wholesale prices for a raw product must necessarily result in 10wer retail prices for a processed product What other factors could have an impact on the final retail prices of cotton clothing? If any of the costs of transforming the raw product into a processed product increase, then the retail prices of cotton clothing will not necessarily fall

A Correct This statement properly identifies a weakness in the argument

B The relative prices of raw wool and raw cotton are irrelevant to price changes in raw cotton and processed cotton

C One step between wholesale and retail prices is the operating cost of the retail store If that operating cost has been constant rather than rising it is possible that the retail prices could follow the lower wholesale prices Thus the argument is not weakened

D The argument notes that the wholesale price has fallen in the last year and that though the retail price has not yet fallen it will inevitably fall The argument has already taken the lag into account and is not weakened by this statement

E Harvesting costs are part of the assumed increased price of raw cotton and do not affect current retail prices

The correct answer is A

Reviewer The book Art's Decline argues that European painters today lack skills that were common among European painters of preceding centuries In this the book must be right since its analysis of 100 paintings 50 old and 50 contemporary, demonstrates convincingly that none of the contemporary paintings are executed as skillfully as the older paintings

Which of the following points to the most serious logical flaw in the reviewer's argument?

(A) The paintings chosen by the book's author for analysis could be those that most support the book's thesis

(B) There could be criteria other than the technical skill of the artist by which to evaluate a painting

(C) The title of the book could cause readers to accept the book's thesis even before they read the analysis of the paintings that supports it

(D) The particular methods currently used by European painters could require less artistic skill than do methods used by painters in other parts of the world

(E) A reader who was not familiar with the language of art criticism might not be convinced by the book's analysis of the 100 paintings

Argument Evaluation

Situation Comparing 50 contemporary paintings with 50 earlier paintings a book argues that contemporary European painters 1ack the skills of earlier European painters The reviewer endorses the book's thesis by noting that the contemporary paintings are not as skillfully done as the older paintings

Reasoning What is the flaw in the reasoning ? The argument is based on two samples of 50 paintings each but there is no evidence that the samples are representative It 1s quite possible that the book presents and discusses only those paintings that support its argument and ignores those paintings that do not support the argument The reviewer accepts the selected samples of artwork without questioning how truly representative they are

A Correct This statement properly identifies a logical flaw in the reviewers argument

B This statement critiques the narrow focus of the book not the reasoning of its argument which is endorsed by the reviewer

C This statement shows that the reasoning of readers may be flawed if they accept the thesis on the basis of the book's tide it does not point to a flaw in the author's or the reviewers reasoning

D The book compares two groups of European painters Comparisons to painters in other parts of the world are irrelevant

E This statement focuses on the reader's possible confusion not on an error of reasoning on the part of the reviewer or author

The correct answer is A

A computer equipped with signature-recognition software which restricts access to a computer to those people whose signatures are on file identifies a person's signature by analyzing not only the form of the signature but also such characteristics as pen pressure and signing speed Even the most adept forgers cannot duplicate all of the characteristics the program analyzes

Which of the following can be logically concluded from the passage above?

(A) The time it takes to record and analyze a signature makes the software impractical for everyday use

(B) Computers equipped with the software will soon be installed in most banks

(C) Nobody can gain access to a computer equipped with the software solely by virtue of skill at forging signatures

(D) Signature-recognition software has taken many years to develop and perfect

(E) In many cases even authorized users are denied legitimate access to computers equipped with the software

Argument Construction

Situation Forgers cannot duplicate all the characteristics that signature-recognition software analyzes including the form of a signature pen pressure and signing speed Computers equipped with this software restrict access to those whose signatures are on file

Reasoning What conclusion can be reached about computers equipped with this software? The passage states that the software detects more characteristics in a signature than the most accomplished forger can possibly reproduce Thus skill at forging signatures is not someone to enough to allow gain access to a computer equipped with the software

A No information about the speed of the analysis is given so no such conclusion can be drawn

B Since the passage never mentions banks no conclusion can be drawn about banks use of the software

C Correct This statement properly identifies a conclusion that can be drawn from the passage

D The time required to develop the software is irrelevant to the argument

E Since errors are not discussed any conclusion about errors is unwarranted

The correct answer is C

Start-up companies financed by venture capitalists have a much lower failure rate than companies financed by other means Source of financing therefore must be a more important causative factor in the success of a start-up company than are such factors as the personal characteristics of the entrepreneur, the quality of strategic planning or the management structure of the company

Which of the following if true most seriously weakens the argument above?

(A) Venture capitalists tend to be more responsive than other sources of financing to changes in a start-up company's financial needs

(B) The strategic planning of a start-up company is a less important factor in the long-term success of the company than are the personal characteristics of the entrepreneur

(C) More than half of all new companies fai1 within five years

(D) The management structures of start-up companies are generally less formal than the management structures of ongoing businesses

(E) Venture capitalists base their decisions to fund start-up companies on such factors as the characteristics of the entrepreneur and quality of strategic planning of the company

Argument Evaluation

Situation When venture capitalists fund start-up companies the failure rate is much lower than when the companies are funded by other means The Success of start-up companies then may be attributed more to their source of funding than to any other factor.

Reasoning What point weakens the argument? The argument concludes that the source of funding is the single most important factor in determining the Success of a start-up company But what if the source of that funding venture capitalists considers other factors before making its investment? Venture capitalists may evaluate the characteristics of the entrepreneur as well as the companys strategic plan and management structure before deciding to fund the start-up company If this is the case then the most important element in determining the success of the company cannot be said to be the source of the funding

A The responsiveness of venture capitalists is a point in favor of the argument not against it

B This statement about the relative importance of strategic planning and the personality of the entrepreneur does not weaken the argument because it does not address the issue of financial backing

C The argument concerns only successful start-up companies so high failure rates are irrelevant

D A comparison between start-up companies and existing businesses is not relevant to an argument about the funding of start-ups

E Correct This statement properly identifies evidence that weakens the argument

The correct answer is E

Questions22-23 are based on the follwing;

Half of the subjects in an experiment-the experimental group-consumed large quantities of a popular artificial sweetener Afterward this group showed lower cognitive abilities than did the other half of the subjects-the control group--who did not consume the sweetener The detrimental effects were attributed to an amino acid that is one of the sweetener's principal constituents

Which of the following if true would best support the conclusion that some ingredient of the sweetener was responsible for the experimental results?

(A) Most consumers of the sweetener do not consume as much of it as the experimental group members did

(B)The amino acid referred to in the conclusion is a component of all proteins some of which must be consumed for adequate nutrition

(C)The quantity of the sweetener consumed by individuals in the experimental group is considered safe by federal food regulators

(D)The two groups of subjects were evenly matched with regard to cognitive abilities prior to the experiment

(E) A second experiment in which subjects consumed large quantities of the sweetener lacked a control group of subjects who were not given the sweetener.

Argument Evaluation

Situation The experimental group in a study consumed large quantities of an artificial sweetener and afterward showed lower cognitive abilities the control group consumed no artificial sweetener and revealed no cognitive decline An amino acid in the sweetener is said to have caused the decline

Reasoning Which point best supports the conclusion? After the experiment the two groups showed a difference in cognitive ability According to the passage it could therefore be concluded that this difference was the result of the experiment What unstated information would support the conclusion? Matching the subjects cognitive ability beforehand eliminates the possibility that one group may have had lower cognitive abilities to begin with The similarity of the two groups prior to the experiment supports the conclusion that an ingredient in the sweetener is responsible for the difference afterward

A The smaller quantity consumed by most people is not relevant to the findings of the experiment

B It is logical to assume that all members of both groups were consuming some of this amino acid in their maintenance of adequate nutrition' for themselves If the amounts consumed were small the conclusion is unaffected If the amounts consumed were large the conclusion that this amino acid causes cognitive decline is weakened not strengthened

C Safety determinations by food regulators are irrelevant to the outcome of the experiment

D Correct This statement properly identifies evidence that supports the conclusion

E A second experiment without a control group does not strengthen or weaken the first experiments findings

The correct answer is D

Which of the following if true would best help explain how the sweetener might produce the observed effect?

(A) The government's analysis of the artificial sweetener determined that it was sold in relatively pure form

(B) A high level of the amino acid in the blood inhibits the synthesis of a substance required for normal brain functioning

(C) Because the sweetener is used primarily as a f00d additive adverse reactions to it are rarely noticed by consumers

(D) The amino acid that is a constituent of the sweetener is also sold separately as a dietary supplement

(E) Subjects in the experiment did not know whether they were consuming the sweetener or a second harmless substance

Argument Construction

Situation The experimental group in a study consumed large quantities of an artificial sweetener and afterward showed lower cognitive abilities the control group consumed no artificial sweetener and revealed no cognitive decline An amino acid in the sweetener is said to have caused the decline (The same as the item above

Reasoning How can the effect of the sweetener on cognitive ability best be explained? This question asks the reader to consider only the amino acid in the sweetener and the lower cognitive abilities found in the experimental group What could link them? The fact that the amino acid inhibits a process necessary for ordinary brain function would explain the decline in cognitive ability found in the experimental group

A The pure form of the sweetener does not explain the experiments results

B Correct This statement properly explains how the amino acid might cause the lower cognitive abilities

C Consumers failure to notice adverse effects from the sweetener does not explain the sweetener's effect on the experimental group

D The availability of the amino acid as a dietary supplement does not explain how consuming the amino acid in the sweetener produces lower cognitive abilities

E The use of a standard experimental procedure does not explain how the sweetener affects cognitive ability



Politica de confidentialitate | Termeni si conditii de utilizare



DISTRIBUIE DOCUMENTUL

Comentarii


Vizualizari: 3041
Importanta: rank

Comenteaza documentul:

Te rugam sa te autentifici sau sa iti faci cont pentru a putea comenta

Creaza cont nou

Termeni si conditii de utilizare | Contact
© SCRIGROUP 2024 . All rights reserved